Ap English Test

  • November 2019
  • PDF

This document was uploaded by user and they confirmed that they have the permission to share it. If you are author or own the copyright of this book, please report to us by using this DMCA report form. Report DMCA


Overview

Download & View Ap English Test as PDF for free.

More details

  • Words: 25,376
  • Pages: 56
r9l:

t lslr 1)ll)vud

p.raqleoB e ssed lq8gu auo sllH aqr uo lno 'a8e1g,r aql urory deme 'cpsual)erer{l sll'puelsr ar{l uo sprrq /![aJ pue sP aJa,/vr aler{l leql osle lueaur ralezrt uado 1o

-uralsrr Jo alsel aql pqII qrnur ra^au aql'saqrnbqu€ ou araM arar{I'lsaroJ-aw s€M lsal aql lnq'ls€o) tpJou aq1;o sadols uop€^Illn) a)€Jral;o saqcled rvra1 e prrp ,{,razrps auros aJaM araq; 'ut3.nn se1vt'lI Jo sa pue /Ioor{DS r{sq8ug ue uerD Jaldaals reJ Sulqlawos paw8eufi p€q I '{Jor*'ryou.'1lort ol arueJ auo alar{1v\ luawalDas puad pasodu4-Jlas Jo lros e se-1r papre8a'r pu€'puelsr aql paqleol sralspur pue sAoQtllog (Sl) 'ase.rqdered aldurrs e ro;. 8u1re.u sace; luelcadxa aql se oru ol >laarD qcnru se lsnl ara^r +erD slura+ Jo [[nJ s)ooqlxa] Jqquar)s uerlraruv atu Surrq plno,lr daqtr uaryo pu€ lpua s,uossal le sselJ qred aqt rc1 @1) Jo lno uraql 8uq1a8 alqnoq peq 1'rer e;o sarueu qsq8ug aq1 1q8ne1 1;r 1pau,r,r.ed daql'uraq1 qlrrra d.4aod s,urduoda IooqJs aql pear o+ pal4 I JI 'aJuarJs

dr+D IIe 'puel$ aqr yo dpoq aql'trpde eBE 'erud1apelq6 latoH aql'loor{rs uor-ig p se '1de dlensr,L lsnf ueql aroru ssaua{Il p

1

;o 'apere; u€r-q+ulro3 aleuro s1r 1o alrds tn ' pue q8ry sda.rols leralas Sntpp q 3uo1 .i e :lr punore sa8ellor aql paJJeMp pue aFe suDlslno aql uo pools'adecspuel aql qtl arLL 'a1dura1 Drroq e ur qpr w a|'uoq uo soxeryd lp sE'srnoqreq o1\{l aql Ic rcau Ialoq uelpreMpg->laarD asaqo ue se.\r a,r,r aJoJaq 3uo1 alqrsrn'salosar{a oml ara-\t al

dlpnba'rar{lo

'sJnoqJer{ IIeUJS lla ro;8tr.ry1dra,ra pue 'arnl€Jalll roJ Suyqlou parer daql punoJ sesnor{ a+rq,vr-,trous to uopeJaurolSSe re 'qq13ug lnoqe cueru8erd dlssayqlnr araM daql 1ng ra,ro aql'rauJoJ auo olur e plroM ar{+ sdoq ueql asroM ou puE Je$aq ou ara,r,r sdoq (99) ^eI'lsaM-quou 1nq Surqlou :pale-\u peq proJlII,^I ueql 'a)uaps'sanoo'sau1d pue'palsa88ns ur aql sa^lasuaql . pallqewrlm sel'v\ puel$ aql Jo st{lual-au]-\ pr€puets raq81q rct e Jo dlerrurapece ara.tt'sarlrsrairlm

orq dpo qlrM drluno) e Jo surIlJI^'srals€ur arD dlurepa3'uouwaure8y pa11DI PrlsatrwaldlS araqrur l,rrort quou 1oo1 e ,{po (sauq rvrorrell-uo1g dlpasoddns (og) uo tm;) IoorIDS Surpreoq e ur 3tr.ntrlee1 lnoqe pmsqe

qruuuaar8 se lq8q se sautd ueauerralrpay\ qlr^a para^or ara/vr sIIn{ sll asnPf,aq'

aJeJ se.u dlneaq sll 'lI raqruarual pup dru lnqs I uar{,r,r dezvr,e qlearq dru salei 'eas Stmrarra lsdqlarue ue rn aler{,rl. {lplq : a>III snua1 raprm Surleog'U rrtes lsrq I ual{ qlearq dru Iooi lI 'InJrlneaq dlssaluo$a

uala

dllueseald Sr4qpuros se,vr arar{l qlIM ulSaq oJ 'loor{rs uordg prol arp Jo a)uelque rrqoqdorlsnelr arll adeosa ol $llez* ro; dpuanbary oB o1 paJroJ sem I

'sralsuour lou'sqdurdu (gg) dq sem 11 'paluneq s€.rr lr JI'puEIsI arp paqf,nol ra^au ppq reac 'plro1!t aql uI apnlllos )IproN lsoru aq+ 'auaa lsel aql se1v\ lI'apnllos .{q paqruSeur'paur€{ 'paleyosr a.raaa. daql ;1 se'aruecgru8rs alqeltmoJreun ue uo >Iool-Molaq re; anboec 1u€lsrp e 1o asdurq8 e (99) 'qled rrtau e;o dtaaocsrp eq+'a{uqs e;o a8essed aql sluala Ileurs lpr{.tr pue 'u€ur aroJaq }sorule -pauaddeg ar{l aroJaq plro^! arp sem lI 'prp dlarer dra.t 'arfiqJeur

sertt 1r-Bunureq;'anbsarnlcrd'dUard lmL lanqealpe rarDo ou se1!{ arar{I'lryqneaq se-u 'alal3 ol derure 3ur-qo1ar1s uea8ay ar{l }o uasaFi aqt qlnos aql o1 logeladrqlre pa>1ead.{1}uas v ol lpools pu€lq aql >looJJ asor{llr uI'sLttPJUflurul paxr;1ear8 e'lsam pu€ qlJou arfi ol :tlaslr se alqE. se adecspuel e Jo Ja+ual aql ut pue snsauuodtr /suaL[tf Ir Jo puelnelu aql uo sallru xIS lnoQe rarueals ileurs € rn srnor{ Srrqzzep lq31a .{e1 sov

auo lnq 1raraq1e3-utsar € ro '7o33e1a8ng e Surd'r.rec ueruo.lv\-luesead pazrtoq P Jo'sleo8 palllq-azuorq Jo (sr) (Burzet? ou selvr aJar{l raununs ur) pog Jalurm slr{ pue

Iryarel p L '$T--s asooq3 lnod g1-1 suo4sanb ol sJamsu€ 'a8Bssed Surrn,rolo; aq1 ;o Surpear

.p.ro Surpuodsarroc aql uI q:ea IIIJ uaql pue uo4sanb qrea ol ramsue lsaq aql asooL{)'uraod.ro a8essed ragy.aldls pue ,ur,ro; ,lua+uo) rrarp uo suoqsanb pu€ s)rom dreralq ruoq suollJalas Jo slsrsuor uollJas slql- :il

rnoq

I-aruII

INOIIf,IIS NOIIISOdI^IOf, ONV ITUNIYUSII-I HSI-IDNII

instead it was far tougher. The crowning irony was (80) that this obsessive industry, this mole-Hle bhndness to their nafural environrnent, was what was considered to be-so tWrlalty English about the system. perhaps to Greeks, made blas6 by living amongihe most beautiful _ landscapes in the world, there was nothing discordant (85) in being cooped up in such a termitary; but it drove me mad with irritation... Soon I took to the hills. None of the other masters even stirred an inch farther than they needed to, and the boys were not allowed beyond the cheaaux de frise" (90) of the high-walled school grounds except on Suniays, and then only for the half-mile along the coast road to the village. The hills were always intoxicatingly clean and light and remote. With no company bui mv own boredom, I began for the first time in my tife to " (95) look at nature, and to regret that I knew its linguage as little as I knew Greek. I became aware of ,to."r,birds, flowers, land, in a new way, and the walking, the swimming, the magnificent climate, the absence of all \afhc, ground or air-for there wasn,t a single car on (100) the island, there being no roads outside the iillage, and aeroplanes passed over not once a month_thes" tt irrg, made me feel healthier than I had ever felt before. I began to get some sort of harmony between body and mind; or so it seemed. It was an illusion. * literally a "horse of plank" or a wooden horse Excerpt from

Copyright

@

Magus by JottnFowles (pp. 52-53). 1928, Dell Books.

(A) "Phraxos" (line 1) (B) "landscape" (line 3) (C) "mainland" (line 2) (D) "Peloporuresus" (line 3) (E) "desert" (line Z) 2. The narrator's first impression (lines g-L7) was best emphasized by his use of

(A) pretentioushyperbole (B) elusive metaphors (C) metaphysical speculations (D) whimsicalonomatopoeia (E) symbolic similes

T

(A)

describe the beauty of the sea and the

(B)

verbalize his anticipation of the representation of the "Corinthian facade,, reinforce the contrast between intellectual pursuits and nafural passions provide a pedantic explanation for the lack of towns on the island analyze his affinity for the physical

(C) (D)

(E)

TRACKING THE AP ENGI-ISH I.ITERATURE IXAM

mountains

surroundings

4. The word "obese" (line 23) refers to the (A) proximity of the hotel to the water correlation between the size of the hotel and the island (c) analogous comparison of a taxi to a hotel (D) intrusion of an ostentatious manifestation of the modern world on the enticing beauty of the island (E) concentration of buildings on the ,,north-west,, section of the island (B)

5. The second paragraph helps to establish the

narrator's

(A)

acceptance of the contrast of civilization and

(B)

uneasiness with his first impression of the

The

1. The word "itself" (line a) refers to

I52

3. The tone the narrator reflects in his description of the island primarily helps to

nature on the island island

(C) understanding

of the significance of the

facade of the buildings in terms of his future

(D)

(E)

experiences on the island perspective of the conflict of his inability to escape the trappings of a proper education critical analysis of the island,s 6eauty

6. Thespeaker establishes the tone of the passage in the fourth paragraph by

(A) (B)

describing the feeling of solitude the island evoked comparing the isolation of the village with the absence of

wild life

(C) reflecting on the island,s ability

to permeate his

(D)

understanding of reality associating his life in school with feelings of

(E)

depression stressing the pedagogy to which he adheres

t9tl

I rsil 1)lllvud

aloq.radlq (s) uorlrlada; (c) dra8eun (r) aqdo.4sode (s)

dpored (v) aql dq pazrseqdrua sr.ra>1eads aqr Jo

xopered (A) loqdepur (q) uo4crp (3)

;:j;}tr

apnlpiJfil ,,

AI'III'II'I (s) AI pue m (a) m pue II o) ru

($l

pue'II,I il

Id1IJXA Surzvrollo; aql Jo IIE apnlrur a8essed slql Jo auraq+ arp a)rofurar dpq reqr s+uaruala eql .gI

pue

I

(g)

(v)

puelsr aql uo Stmlreal;o ruruprmuor ar11 .nI

Euelrqegur

rn apr4rlos Jo sluaruoru srq dolua;a8uol ou ppo^t. ar{ (g) $laarD aql pue qs1gug aql uaa^qaq lno >lealq ol lnoq€ r€,vr p se,vr araql (q) sIF-q aq+

1o

srmreaur aru+ a,il

aurof,aquoos

*J"i?fl:il:i:?o:J:u, ,r,

plno^ puel$ aqluo

s/pu€Isr aq1 yo dqlede aql Jo acuenbazuor

.11

(og

aqD ,,ssaupullq a{rl-alour,,

aql

.I

sarldun Jo asn s,ra{eads

aq;

.61

drcarp

Jaql€am

plnom s8urlaa; snoruourreq s,.raleads

aq+ (g)

luau4uroddesrp (S)

aril

uonpprda4 (CI) uorsryuor ()) uoqela (g) uorre4sruJ (V)

(V)

sarrdun (7g1 aulf,,uolsnlll ue sewtL, .UI

luatlgdurr (A) prua8uoc (q) rrrcqrede (J) pm8uel (g) rruorel (y)

uer luaruu'rsse srq

a;q o1 qreordde.rlaql ale sJalseru pue sluaprys aql (A)

0,"];;:;il;?tj:;i;X5ff ,

pnuanbasuocurdl8urxaldrad (A)

alenbapell8urpatuorsp (A) snong.radnsdpnopaz.razro (3)

aqr+eqrqde.r'e.red"",",Jlit;l:#";Ti1"*'",ff ,, >Ise+

.11I

uralsri's

Fuo4€Jnpa qsplrg aql;o s8unuoouor{s

IrourJnl Jarrur +uo4uoc

rn paluarro

aq1

s+uapn+s aq+ Jo qsaJa+ur crdodur_aq1

)rer{rredlSurdouue (S) arcorpaudilercqradns (V) sr

soxerqd uo uorleJnpa

1eq1

sagdun ra4eads

arll

.g

a8pa1mou1 Jo uoqecrldde

Jqueruor aqt ueqt ragle; crleur8erd aq+ o+ uolluaDE JIaq+ sparlp loor{Js aql (cD s8urprmor*ns rraql o+ snoJ^llqo ruaas sluapn+s aq+ pu€ sralseru aW (f,) Ioor{Js aq+ uI s+uapnls aql ol lse4uoo pcrxope.red e sapr,rord puel$ ar{+ uo sTaarD aq} Jo apntqle aq+ (A) e yo

aruolrda aqr q rooqDs

"-Tjt::ffiIfri:*

,o,

Jdg3yg Sqzr,rolloJ aqr Jo II€ errofurar ol sdlaq (gg auq) ,,dre1rru.ra1,, roqdeljru iq1 .ZI ;o n.r, aql

pue dlrnb4ue;o a.raqdsou4" .',n

"r"3iHJr?i,i" papnxa

1r;

puel$ aql leql Jnapu€r8;o a8eun ue apr,ro.rd (q) yo d.ro1s1q arfi uo

q/ur

yo

"r,r"r,o*oJit|l3rT"" ,r,

slua^a sno44da'ns aq+ roJ r"o""f"tfij#Hro (s) uorssed;o esoql pue palalur Jo slrnsmd aq+ uaamleq lDrruuor Sur.dyraprm ue qsrlqelsa (y)

uouuraure8y pue er+saurua/13 oi

,".rJrt;ffi

.,

Ouestions 16-27. Choose your answers to each of the following questions based on careful reading of the following poem by Christina Rossetti. Passing away, saith the World, passing away: Chances, beauty and youth sapped day by day:

Thy life never continueth in one stay. Line Is the eye waxen dim, is the dark hair changing to gray (5) That hath won neither laurel nor bay? I shall clothe myself in Spring and bud in May: Thou, root stricken, shalt not rebuild thy decay On my bosom for aye. Then I answered: Yea. (10) Passrng away, saith my Soul, passing away: With its burden of fear and hope, or labor and play; Hearken what the past doth witness and say: Rust in thy gold, a moth is in thine array, A canker is in thy bud, thy leaf must decay. (15) At midnight, at cockcrow, at moming, one certain day Lo the bridegroom shall come and shall not delay: Watch thou and pray. Then I answered: Yea. Passing away, saith my God, passing away: (20) Wrnter passeth after the long delay: New grapes on the vine, new figs on the tender spray, Turtle calleth turtle in Heaven's May. Tho'I tarry, wait for Me, trust Me, watch and pray. Arise, come away, night is past and 1o it is day, (25) My love, My sister, My spouse, thou shalt hear Me say. Then I answered: Yea. 16. How many speakers does the poem directly present?

(A) (B) (C) (D) (E)

18.

Lines G-7 suggest that

(A)

(B) (C) (D)

(E)

(A) (B) (C) (D) (E)

Realization of death's inevitabilify; fear of physical decay; passive acceptance of what

(B)

Nostalgia for the earthly world that must be left behind; fear of physical decay; welcome acceptance of the afterlife Realization that death will come before one's ambitions have been achieved; dismay over the visible signs of physical decay; supplication for the healing powers of divine

(C)

TWo

(B) (C) (D)

(E)

I54

I

flowers highly prized for their rarity which bloom briefly and beautifully and then die spices which add flavor to food and, metaphorically, to life leaves kaditionally woven into wreaths to honor poets traditional symbols for Homer and Ovid respectively traditional symbols for true faith and pious conduct, respectively

CRACKING THE AP ENGI.ISH LITERATURE IXAM

Line 6 LrneT Line 13 Line 14 Line 21

(A)

cannot be escaped

intervention

(D)

17. "Laurel" and"bay" (line 5) are allusions to

(A)

death there is a natural cyclical pattem of renewal that the principal narrator has forsaken the principal narrator is gravely ill and certain to die before the spring

20. \A/hich of the following choices best characterizes the speaker's attitude in each of the poem's three stanzas, respectively?

Three Five

unbeliever although the World has regenerative powers, the principal narrator of the poem does not paradoxically, life can sometimes emerge from

t9. \A/hich of the following lines contains an image NOT echoed closely elsewhere in the poem?

One

Four

the principal narrator is faced with a choice between the afterlife that true faith offers or the physical corruption that awaits the

(E)

Sorrow and mild surprise at the arrival of early death; deepening awareness of death's certainty; hopefulness for a place in the

afterlife Acknowledgment of death's inevitability; understanding of the need to prepare oneself; happiness at the prospect of union with the

divine

ggll t IsiI Dtllvud

pueqsnq pa^olaq raq aruoraq

IIr.{ or{^. ueur arp paru dgeryua^a ilwr aqs salsatldo.rd pos s, rolerreu pdrcurrd aq; (g) oleq+_

buruJoru pue lq8ruprur uaamlaq eurq atuos lp alrJre grzvr ra8uerls Lr a pue snorralsAu e papro^e aq louupJ roleJJeu ledrcurrd aq+ s+JaJr4 +sJtl ssau[r llpeap e uaqM rnoq aql

a^rIre rrr^ er

aqr* o""T,fflJT;::r"#T;:*

rnoq leq1vl ,{11texa "q lllvr leql ;urmotr{ Jo &uJe}rattm llerus aql "1rd."p 'aruoJ IIrM rnoq leuq,,ro1errn.,

fndp.+rd

leril

+sa88ns

"r'trl

(c)

(r) (g)

(v)

9I pue gI saur.I

"ro8rn;o ssol pue

.gZ

"^",s q+eap Jo saruar'

ar'

Jo

uo'errFT::,n;5x;

aql raqury ol VJrqzvr. punore roqdelaru Ierluar t1ut":+l sasodord ezuets q)pa qrrqm .:l?ttr Aq uorssarHord aql sanurluoJ ezuels prlrD aql ,,'ta+srs,, pup,/a^oll suJal arfl Burcnpor+urlg .sezuels asoql dq paJuelpp smar^ aql sleada.r pu€ surrrJJeor ezvels prlq+ aql . sr

'sezue+s rarlr€a eq1 yo dra8eu, nq1 e*oq"" Xg .IeMauaJ se pazrJalJeJerlJ pea+sur

{e:ap ot poua{{ uaaq per{ rarlrea lEqM leql leql slseJar lnq,r(raSeurr ,in unif. o^{l lsrrJ aq+ Jo rpnul saoqJa ezuu+s prr-q+ ar{I

u",re+ aq

allu4s

luawqwefua (3) pra88op (g) aqdo4sode (y)

(c)

yo aldurexa ue aprzrord

())

(s)

aql;o .{ra8eur ,{ep

.-_:-Tp"naql raqlaSo; -btrtppam salea.t,r ezuelspriq+

aqf (V) zsezue$ )L'o)as pue lsrg aql Jo l€q+ ol ezuels prlrfi aW Jo drqsuo11e1a.r

g pve Lsav_I .ZZ

aJII Jo sassaJo"rd prnleu pue lualrs aql lq paAo4sap r(1enpe.r8 Stnaq sI dpoq,,ro1nrrlr, (A)

'ezuels lsrrJ ar{l;o d;a8erur arurfurrds

?I:f"i:*!

(A)

roqdepru paxnu (q)

aql

ezue+s p-rroros

3uo1 (y)

,]atLno_,, ezvetsorilit"Trofr ,a

G)

os Arabeurr

lq pauorrsanb uaql pue "rJrAHrH;?; parue^pe ara1l^ qrnlM saurar{l aql sdolJrrap raqlrnJ ezue+s prr-q+ aqr d.ra8eun s1, q8norql 's^rarl Surrafirp ,sezue+s rarlrea asoql Bulrcuo"a, lq"i"q1

u'r

(q)

3] ";:j::5"Tilyjfi aJrI AFDrpa (S) aseasrp

o+

BuBe

."r,r"1"nrnq"

Alalernrrp lsour sluarualels Buraa,olloy "ql aqr .rZ Jo qrlqr\A

rr"'^ltT,

rnrvr

pardrur.i:iii.f;,T,:1l", rcl dqlromun

aJIIra+Je arp Jo

+r Japual IIrM . 'uorlJe au:os sa{el Jo+eJJeu aq} ssapn,q"rU^ +uaruala alrlrn4sap E surcluoJ Inos s,rolerreu

aseasrp a>IrI_JaJUPJ e ro

'raJue) dq paurnsuoo Buraq,, srporu lq ualea Buraq sr

ipoq s,roleJreu

()) (S)

"ia;,,,roinrreu (y)

aq+ leql dldrur o1 papua+ur sr (g1 auq) ..f,e;cie aqqi q sr qlow e,, uraod aq1 yo 1xn1.roo arfi uI .IZ

26. Irr context, the word ',spray,,(line 21) most nearly means

(A) tree (B) blanket (C) a small branch (D) a liquid mist (E) a holy spirit

I56

;

(RACKING THE AP ENGTISH TITERATURE EXAM

27.The grammatical subject of the sentence that begins at line 24 is

(A) "Arise,, is past and to it is day,, lli ..gsl. My love, My sister, My rpo.rr",, lSl "thou,, (D) (E)

lsrr

I IslI l)[)vud

-.raaq Sur.rnoqq8rau aq1 o1 der*e sdrls aq;r'saqtaarq qsnld pal ur rvtad Jar{ ol alpnord sJtr l slroJsa orIM (09) orar{ +oo}-xrs aql apqaq aoM 'ace1d luacap e;o adoq

aI$H raq sa^eal qJIqM',rallereql € qll1lt luas os

duorr

(A)

eur8naz (q) eraodoleruouo (3)

wduolaur (A) aloqradlq (y) Jo

aldurexa ue sr g auII

ul,teln+Il, pror! aril

Jo asn

aqJ '09

(rg-gg saqD ,,srapuaJJo qJns roJ

sn8ry;o

sada aq1

seq,, G)

(LV-qV sardlJ) aql,, Sutr.rqs-1nos asrnoJsrp rW ,,ado1g Jo

aserqd agl ur dlpalurod lsour ralJer€qf, uelr€+IJoqlns s,alpnord sIlN '62

'pa>lcaduaq

sr doqsrq aq+'+JBJ

u1 'pqrrrds s8tqp woq rnelsqe (0/)

uala lou IIII\ pue'sluarua,totu sn{ II€ JaAo raanod;aq sar{f,laqs pu€'uonnurop aruoq qDns rDI1v\ paqsr}€s lou sr arpnord sJI tr +ng d13r41gm, pr( 'dprelrmlol lou Jr 'rar{ ol pauopueqe aneq lq8gr alpnord rC f,4saurop s8qqI

2duo,n;o asn s,Jo+€JJeu aql Jo aql Jo i{Jr,qM '82

'1e sr-{l sr roN'uorr Jo poJ e q}IM salnr pue'prol relnlp (9/ rar{ JaAo aurardns salnJ ar{s Drlsaruop sJailEru rq leql sI aurT qlnr+ aqJ 'ssaurddeq s,pueqsm{ raq ol r{Jnur spp€ aqs san+Jr^ raq il€ qll^{ leql >FryD louueJ I II-qs rnq'alpnord srtr41 lsure8e pJoM p aqlpaJq ol uollualur du 1ou sr 11

sr suo4dr.rcsap SurzvrolloJ

3q1py;o pealsur'doqs

(SZ)

lp auror{ dddeqlaq ul'ayr.tr s,uoJeapr{JJe aqJ

(Z-t saqD ,,alpnord sr;41 lsure8e prolrt e aqlearq ol uoqualu dru 1ou q lL, (V)

q

(0€)

pue (g;) sr'seq aqs dezvrs;r dezvrs s,d1+uer5 srry lng 'ace1d^s€a

(ff-Of sauq) ,,paloaduaq sr doqstq aq1,, (g)

aq1

(gt)

auol Surruoraq uI prnru u-l!^.o rar{ ssardxa pue'>IueJ rar{ ;o sa8aprurd ilry aql alunsse ol rvroq smouT'pealsurnld

(6I_8I SAqD ,,1r arrnbce 01 ualr4s d11n;ssarcnsun +ou s€r{ pue'rarvlod sanl€A aqs ssallqnop,, (J)

palerrdordde leas >peq

(0r)

'alnJ s/ueuroM e Jo s+null ai{+ aq pFoqs +€tlx\ sMouT aqs $q (02) 11r a,rrnbre ol uaAIJ+s d1p;ssacrnstm lou ser{ pue'rarvrod sanlel ar{s ssallqnop ldreqs $Iool rar{ rou'pnol ralau sr aJrol raq laJuarpaqo 1o uralled e sI ar{s plrom aq+ aroJaq lpueqsnq rar{ saru€r{s ra^au ar{S 'luarlJauaq pup

lou,, (v)

'qleqq€S aq+ Jo uoqelrasap e a'rr8ro; arpnord srIN IIII!{ durouoca ro rnapuer8 roJ uala lou lnq 'qr11d q8ry e 1e 1da1 sdezwp aJ€ slerour ar4;r parnoord aq o1 dlpreq ale sa8e.u, ,vrol uo laal xls 'ro;'paloopa,lo aq.{eur >laam ar{+ w ssauua>lurup puorserro 'srapuatJo r{rns roJ sn8ryyo sada aql seq alpnord srtr\l'asn srq ol

(92)

;o sar(a arD ut JaaJeJ srq uaaq s€r{ se Iryssarcng'rrlodsap $ aqs pu€qsnq rood raq ol lnq'11e o1 arrqeluor{lne dpnlgqeq sr dpel snIJ'alpnord srtr tr os +oN

(OhSg saurl) ,,depuns aq+ uo luaurdoldura Surlaaqr due uorl aJuaw+sqe 1ta1.rad e,, (A) OZ-SZ sauq),,1q8r'r ra,tau sr aq aJrzlt stq;o sada arp uI ruaas plnom lL, (61)

alduexa ue

ar{s

qdwape uala r{1arer aq'paept4 lwlr{ ruorJ passed lasturq Surpuayap;o adoq ilV'lqBF ralau sI aq atrm srq;o sada aq+ uI ter{+ uraas plno^{ 1r'pllozvr eW

(8 ar4D

u.I!^.or{s sr

$

1o51 'ado1g IIAI

3uo1 seq

qceo.rdde lsaJeau aql sacnpord uorssnrrqns,, (3) (ff-gf sauq) ,,>luer rar{ ;o sa8aprurd IInJ aql aurnsse ol rvroI szltorDl (g)

,,uorururop aruor{ qJns qlIM pal}s4€s

ar{s sr

-;1as

(q)

GZ-1Z saurl) ,,aread

o1

dpo

}o asrnoJslp 8urr.r4s-1nos aql Jo pealsur'4,red s,lua8a5 ar{l uI ileaqlaa.la's e yo dauoq aql o1 Surualsq uaaq arcqo1 prmoJ sI orl^l preuasnor{ +uearrar arD apqaq aoM'IIe sapnlrur r{+pqq€S aq} Jo ssar4)rr1s Surlesuaduroc aq} 'pueqsnq rar{ pu€ dpureu sluplras Jaq'papualxa lou are sassarp ,r,rol pue uopedlsslp aql woqm ol toor awwo raq raprm asorD ro} Alaleun+roruo ^eptms luarudoldua Surraaqc due uror; acuarrlsqe pa;rad e pue ?lasJar{ dq pea.r uoruras Stmrarra u€'saJlllas aarql dq ro; pauole 'Iorluo) raq Japun'are >laam aql Surrnp sassarp rlrol pue uorledrssrq 'alnJ uelre+€qq€S Jo aJu€ Jasqo lJr4s e dq sr;1as1r slrtor{s ,{cuapual sr-ql qJlr{zvr uI lurot aqt pue iuerrrom snor8qar e derur uznto rar{ ur $ aqs 'ppor* ar{+ Jo sJauu€ur pu€ dlarcos aq1 o1 asrale 1ou q8noql 'pauoquaur aq lsnru a;rrvr s,doqsrq aql Jo ral]€r€r{J aql uI dlrreqncad pa>Ireur Jaqlo auo 'qe+lP re^a goeo.rdde acead o1 u^{.o srq r{Jlqlvr asnoq u€r lsar€au ar{1. sacnpo.rd uolssnuqns lprp areme sr pue 1uor1erryl1snl

lnq'Uup€ luas

(99

)

'ado11or1

duoqluy f,qsnmol n$a4Jfig Ia^ou aql tuorl ldrarxa up sr uorpalas

dlryareo a8essed

aql 'sJamsue.rnod asooqJ nod aro;aq 3um,r,o11o;

aql pea1 T7-8-Z=u6trs5fr-O

3L.

the context of the passage, the phrase "if not voluntarily, yet willingly" (line 7) is used to show Dr'

La

Proudie's attitude toward

(A)

the duties that the clergy are expected to assume

(B) entering the institution of marriage (C) strictSabbatarianism (D) granting his wife some Power (E) the hiting of domestic helP 32. The description of Mrs Grantly serves to (A) provide another example of the power of the aristocracY

(B) prove that Mrs Grant$ henpecks.her husband (C) imply specific faults of Mrs Proudie Proudie inl t"ggltf a rivalry between her and Mrs not and seen be should women iEj utJ-".t *hy heard

36. In context, the adjective "recreant" Qine t14) is best

interpreted as meaning

(A) unfaithful and disloYal (B) engaging in a Pastime (C) refreshing (D) craven and cowardlY (E) depraved 37. What is the effect of the repetition of the phrase "Woe betide..." in the final paragraph?

(A) It retards the tempo of the prose' (B) It satirizes the fate of the servants' (C) It highlights the drama of the situation' (D) It chinges the point of view of the narrator'

igj

It emphasizes ihe moral consequences of the action.

38. In context, the word "chatacter" (line 31) is best

interpreted as meaning 33. The narrator's attitude toward Mrs Proudie can best be described as one of

(A) pity (B) objectivitY (C) emotionaljudgment (D) sardoniccondemnation (E) jaded disgust 34.

\ /hich of the following

best describes Dr Proudie's

(A) dubious Personage (B) reference (C) antagonist (D) conscience (E) footman 39. Which of the following best describes the effect of the last ParagraPh?

(A) It suggests

relationshiP to his wife?

(A) MorallY devoted (B) ComPletelY servile (C) AwkwardlY tender (D) ThoroughlYuxorious (E) BitterlYtYrannical 35. The author atfributes Dr Proudie's attitude and behavior most clearlY to

(A) ambition (B)

pride

(c) pacifism (D) spirituality (E)

I58

I

feudalism

CRACKING THE AP ENGLISH TITERATURE EXAM

a cause of Mrs Proudie's moral

transformation.

(B) It introduces Mr Slope as an observer (C)

of Mrs

Proudie's actions. It illustrates how Mrs Proudie's religious beliefs reflect her character' speculations about Mrs Proudie's character. It shows how hard it is to hire household servants.

(D) It counters

(E)

40. The style of the passage as a whole can best be described as

(A) humorless and Pedantic (B) effusive and subjective (C) descriPtive and metaPhorical (D) terse and ePigrammatic (E) wittY and analYtical

6il1

I IS]I lllllvud

'1no 1da,ra.s'a1nor1ar.raq ur.raqdosopud rIJuaJ{ € prrv p>1ood rar{ w uaru poo8 uarrala q+rzrt';aq1otr41 sy

Gr)

laTJop aq1 Surdpn+s 'paleSora.radns 'a8pnf aq1 'loll€q Surgurs /1eulJ aql Sur.rnp a8ed 1se1 aq; 'uala lno auror Surzreq'8ueq e qlIM aJrello1 +nr{s ar{S 'arualuas Surctmouord raqlontr a>lII paooJ ralaN

Io4uo) ou per{

sa^oQ @?)

e ([) e (C) e (J) Trads-du € (g) e ruory Surragns ueru e (y)

s€q aq qrlq1!( JaAo saJu€+slunJrlD Jo luIlJI^ ,,>tuEd arD >IIe^.,, o+ apelu aq III.\,'. or{1!^' ueur +srreuluras ssauIIT leurura+

iraru€J q+rm paqlnoru-ualloMs 'lua4ed srq ^tou puv '''salol'sllqeq puosrad'acue.readde'aJr+)eJd 'uorssayord 'tlulq dq'pauroop 'pawoop 'pauoop sem aH :1\^ar^

ualala

aq o1 paqdrur

d1pa4p +soru $ +squap aq+'€zuels puo)as aql

q '€t

auqueru€pe Jar{ o+'alrJop'sJaquraur-1talJ pal'sluaurded dsea srq ile {oo+ rar{ro4

aq+

arl

sunJ's1oo1

-lllJp Surquror'raq drun€

"ql

qDIq&

q

I€rJl ar{+ se Surpear o+ uoqua+le q)nur se sdedlaqloru aql +eql peJ arp aJroJular

dlrapla se rarpour ol salras

1e loodplqrvr du11 aql lsed sarelS '8rmr1stm 14s lluaqed € s+rrup€ aH

sleads lsquap auol larnb d.ra.t aqt 1sa33ns (s)

uorsrJap rar{ saqJear rar{lour aq+ qJrqm q+mt paads aq+ azrseqdrua IIaM reaq lou saop raq+oru aql +€t{+ 1!(oqs

(S

aqD

aq+

azrra+)em{f,

,prea:q1-Ile:q.,,

aserqd

'prara51 Jrls€ru sr-q ruoq 1se1 1e dezu.e 3u1p1 (gg) 'sllg r.uoor uo4daJar arD s€ aJueIIIPp rraql aro)s

(c)

(r)

sarun{J leJrsnur lt{.aN

(s)

'I{JUIIJ arD le pauuIJS qlaa+ .ra1se1d Jo saluorleq IInJ araqa

(v)

aql

'ZV

dlpn8

qe1

alftl

auo (3)

uollenlls.raq dq raq uodn

pasodun saqnp arfi Jo ar€Metm dlapldruoc (g) paraplsuoJ pue paJa^oJun aq S+JEJ lu€^alar aql il€ 1er1tr Srnaas ur palsaralur (y)

aw .,, sppruratr/{ uro{,,sro})ocl;o aurl 3uo1y,,

'aseald '1no

'I7

qlIM

(92)

'sasllra^p€ lsrluap sHl'II€ ]o +sroM 1r 1rd5-JaqJeq 'ra>lzvreq 'ra>1ceq 'sraqd ;6

>Irar^( aql pure /arrpllo^'raq ol Irre) (01)

r""iTf;#j

ilJo u4r{ qsnd pzvr aq5 'peu.ra8ur.; rar{ qrIM

lcads-dg e lcrg'a8ed e surnl'surra.o.r1 raqlot'{ 'q8noo snolrau e srapuar'>1ue1d aql slunotr I >Irap aq+ ssor)€ pal $ ls4uap a^r+mJ aIDII aqJ /+r '1tau paleu srq Surreq lnor{}Im dn sa1oq3 (g)

slua^ul 'luaruPodaq (99)

ry auar8dll uI sl)nrlsur'acrlre,rd leraua8 e se11 'urtoul 1a.rt slal€lopr anr+ aql se'oq A Jo

raq

rapIaI^{

:aJ€J pue arnJ sIH rOJ Saseaslp

"rD

s,11op

'raplart'cruerpau aql sa8pal.r,toupe aqs'Srnzaa.rg (91) 'saal8ap alseqD aql'ldlrJs ur1e1 a1qr8al1 'a1e1d sserq alqepadsar aq1 's8qpd uorl ar{J puodaq are slsquap lsuenrsdqd JJI+s JO arnl e ruoq papuaJsap lssaupaprnurlet rar{ JO

'a4pllo1 a>lrT'alqesuadqpnt umr smouT aqs

dUeC

prmor puB punor'fq

'lsruorldarar lrr8 ualrs srq paser{J aq a,t.oq :saslU duourrlsal aql se aulrq 3rn1se1 1de.ra.e ldarvrg sa8prrq a>l{-aJII sn{ il€'ssalule4 dllaruro4 'trnq sasdurq8 aqs 'qldap raq puodaq ra.r.ag (p7) 'a{o4s lsearq dsea ue qlIM II arunlo1 q8norql 8.4pIIC

sdezvrle +sorule

aq ol raq smor{s uraod aql rn pa}elar IqIJI aq+ pJ€Mol apnlpl€ s,rar{lolu aq+'ll€ra^O

ar{+

as.rnu Surdeld 1ape1s e a41 'paddq-asrnd

aql a^oq€ Jlasraq sJaplsuoJ oq auo (g) ^ ^rel dpq8U oo1 rornf e se sarlrlrqrsuodsallaq 3ur1e1 qlIM pauraruoetm (6) are IErq o1 lq8norq asoql leql sa^allaq oqzu'

(gtl

aql-islql {r€IAtr ranps srt.l JaAo leq fuarrls 'ra11

'an8uol s,urn)rl srq q8norql Ilrrp

8uq1eo sn{ sI JelIoJ asoqm lsrJerfluras e

sY

au17

uolsral umo rar{ Sursodurl

,,'arar{l lq8p'a1q rnod 1no 1rdg,, urrq 'des uaq+'dernte mry o1 'tq urlq padde'4s pW bS)

/surJe ar{+ Surddr'rD 'uM.op padoorvrs ppr{ aqs asn€Jaq 'rreqr dsearm s1q o1 Sur8urlo'1oo; s,a^ol lV >lJ€q >lool rala aqs prp 'srsatuall 's4 I acrN

llgn8

sea,r lqluap ar{l'prom B preaq-Jl€r{ aqs aroJag aqs'dervrduy'sournlol aalp q a4u41o1lo afi7y q8no.rq+ pB o1 pa8eueru drnp drn[ ro1 pa>1o1d zra.arl>I

"raq1otr41

'.e\olloJ leg1 suo4sanb aql 01 sJaMsue asooqJ

pue'd1p1arer

^

uaql

olaq uraod aql pea5'EE=ftTiroJJJa-7fO

44. Which of the following best conveys the meaning in context of "Freezing, she acknowledges the mechanic, welder, wielder/ Of pliers, hacker, hawker, barber" (lines 15-17)?

(A) (B) (C) (D)

(E)

48. The phrase "tasting

brine" (line 22) indicates the

dentist's (A) desire for the trial to be over quickly (B) anger at the falsehoods offered as testimony

The mother thinks of other tradesmen she dislikes as much as she does dentists. The mother thinks of professions similar to

against him (c) shame at the revelations of his unprofessional

dentistry. The mother thinks of the diverse and distasteful aspects of the dentist's profession. The mother thinks of trades which, like dentistry, she recognizes as necessary although disagreeable. The mother thinks of the control typically male professions exert upon her and how in this instance the tables are turned.

(D) fear of being imprisoned for his acts (E) sense of the growing likelihood of a guilty

45 The phrase,

"this dentist advertises" (line

18)

princi-

behaviour

verdict 49. Which word is used as a metaphor for reading? (A) Dreaming (B) Walking (c) Conversing (D) Swimming (E)

Flying

pally suggests that (A) the dentist is unscrupulous (B) the dentist is not professionally qualified (c) the dentist's lack of skill causes him to constantly seek new clientele (D) the dentist is a newcomer to the area (E) the dentist offends the mother's sense of

propriety 46. The poem states or implies which of the following?

I.

47.

II. m.

To a large degree the mother finds the dentist guilty because he is a dentist. The jury finds the dentist guilty. The dentist should be found innocent.

(A) (B) (C) (D) (E)

I only II only III only I and II only I,II, and III

In the fourth stanza, the dentist is portrayed as

(A) (B) (C) (D) (E)

I6(l

f

comicallylecherous

brutally vicious calculatinglycriminal

timidly amorous angrily frustrated

CRACKING THE AP ENGIISH TITERATURT TXAM

50. Which stanza suggests that the mother's treatment of the dentist could be seen as "poetic justice?"

(A) (B)

(c)

(D) (E)

4

s 6

7 8

51. "Nice Mrs. Nemesis" (line 47) is an example of

(A) understatement (B) hyperbole (C) irony (D) personification (E) onomatopoeia

l9ll

I Islr l)rl)vud

.NOTtf,iIS

'OS OCI

NO OD ION OCI OI CI'IOI !ruV NOA'II,INN II NOII)gS ,CIg'I'IVf,OI SI SNUI iruOdgg HSINIC NOA CI SIHI NO XTIOM UNOA ))!IHJ AYI^iI NOA I NOIIf,IIS {O CINII

dOIS

(Sl aqD ,,alncBar rat{,/ (Sl au1D ,,raqdoso11qd qJuar{,, (Sf aq) ,,uaur poo8 ualala,, (St aqD /,raglotrAl/, (m aqD ,,a8pn[ aqg,,

aruaraJlpur pIoJ (c) aluEr{s

dpo111puel

(J)

dponIpuel

snornrv (s) (v)

druo druo

JeBuE pal€4sruC

ZaJr^ras ,(m[ s,raqloru aql pJ€Mol apruFl€ s,pod aql saqrrrsap lsaq sarroqc 3u1mo11o1 aql Jo auo qrTqM 'gg e Jo

luau4earlsrlu

uorssayo.rd sTq (S) slalaq sno€qar sfq (V)

IcIiIf, XA suos€al Surmogo; aqr Jo IIe roJ lslluap aq1 ;o saaorddesrp .raqlour aqJ

n

rt

(S) (CD

(r) (s)

dpot (v)

'urnroJap lelcos rol paau aq+ ur poD l€r{1 sa^arlaq laq+oru ar{I 'il sareqs raq JaIIaq 's,aJr€llo^ to asoql ol JEIIurs aJ€ s1v\ar^ s/poD lpql sa^arlaq Jaqloru ar{J 'II 'palecgsr,qdostm rnq plaq dldaap sI poD Jo uorslA s,rarilour ar{I 'I

sH (:I) rou€aruap sr-q (A) lu€lsrss€ sr-q qll/v\ rreJJe sr-q ())

luaued

(s)

(v)

patqns EI s€ sa{el (Sl aql) ,,lno ldaals,, 'dlprrleururer5 's9

a)ual€Arqru€ pasnurv (a) arns€aldslp [nJuroJS

€)

(c) (J)

'tg

aql Jo r{Jrqm qsaSSns

e

23u1ru,ogoy

Jels

pug s,ruaod aq1

ENCLISH LITERATURE AND COMPOSITION

SECTIONII Total Time-2 hours

Question

1

(Suggested time---4O minutes. This question counts as one-third of the total essay score.) The passage that follows is excerpted from Don Delillo's novel Libra, (19gg), a fictional treatment of the young Lee Harvey oswald' who as an adult wbuld assassinate President John F. Kennedy. Read the passage carefully. Then write a well-organized essay concerning the methods by which the author nas portrayed the subject and the substance of the portrait itself' Be sure to consider such literary elements as dictiory

imagery, and point of view.

He returned to the seventh grade until classes bundles on the wicker seats. He jumped the turnstiles ended. In summer dusk the giris lingered near the once. He rode between cars, grfppina the heavy chain. benches on Bronx park South. Jewisi girls, Italian girls (45) He felt the friction of the ridJin his t-eeth. They went Line tn tight skirts, girls with ankle bracelei-s, their voicJs so fast sometimes. He liked the feeling they were (5) murmurous with the sound of boys, names, with song on the edge. How do we know the mJtorman,s not lyrics, little remarkshe didn,t always understand. ThJy insane? It gave him a funny thrill. The wheels touched talked to him when he walked by making him smile in off showers of blue-white iparks, tremendous hissing his secret way. (50) bwsts, on the edge of no-cor.rt ot. people crowded in. Oh a woman with beer on her breath, on the bus e-very shape face in the book of faces. They pushed (10) corninghome from the beach. He feels the tired saltv through the doors, they-hung from the por"ulul., straps. sting in his eyes of a day in the sun and water. He was riding just to ride. The noise had a power and "The trouble leaving you with my siste1,,, a human force. The dark had a power. He stood ,,she at the Marguerite said, had too many children of her (55) front of the first car, hands flat against the glass. The own. Plus the normal disputes of family. That meant I view down the tracks was a form of power. It was a (15) had to employ Mrs. Roach, on pauline Street, when secret ard a power. The beams picked out secret things. you were two. But I came home one dav and saw The noise was pitched to a furyhe located in the mini, she whipped you, raising welts on yor, l"gr, and we a satisfying wave of rage and pain. moved to Sherwood Forest Drive.,, Heat entered the flat through the walls and (20) wtndows, seeped down from ih" tu, roof. Men on Sundays carried pastry in white boxes. An Italian was inurdered in a candy store, shot five times, his brains dashing the wall near the comic_book rack. Kids trooped to the store from all around to see the (ZS) of grayish spafter. His mother sold stockings in lr1cey

Manhattan. A woman on the street, completely ordina ry, maybe fifty years old, wearing glasses ind a"dark dress, handed him a leaflet at the foot of the El steps. Save the (30,) Rosenbergs, it said. He tried to give it back if,lrrt ing he would have to pay for it, but she,d already rurnJa away. He walked home, hearing a lazy radiovoice doing a ballgame. plenty of room, folks. Come on out for the rest of this game and all of the second. It (35) was Sunday, Mother,s Day, and he folded the leaflet neatly and put it in his pocket to save for later. There is a world inside the world. He rode the subway up to Inwood, out to Shgepshead Bay. There were serious men down there, (40) rocktng-inthe copper light. He saw, beggars, men who talked to God, men who lived on ilie trains, dav and night, bruised, with matted hair, asleep in patient

I62 N

CRACKING THE AP ENGTISH TITERATURT TXAM

t9lg

I rstl l)tllvud

uerrrauy

druapery aqa dq ffi02-l66lO lq8pddoS ,", ,"r-Uri|;: xq pasf p"-asar sfq8rrlly pal'lq9g6l O lq8pddo3 '996I ,MoU ,"d"n11 lq paqsrlqndl ;r{b;; ry ruy *oij 1o

sur11o3.rad.re11 l:lrzu. luaruaSuer.re

qle14 eu.1dg-

'suoolleq a{rl asrr sla.\,l.o^ realJ

t4

nod ,*ou puv.slers

rnp.r,

#:ilf.tHflH#n

arenbs ,vropurM ar{J .s,leJ e se u€alJ suado

relorr pue rneaq-.noa

,r",

qlnout ;rr.ro1 (St)

J#:il#fiT#?j:H rea dur ur salour eas

:uals{ ol a)e,vr I .sasor 1wd 1ep lsearq s,raqloru Au uodn >llns oI

,,0,"1?ll,Ttol'Jll j',","#"ffi x,T,",g 'spueq s,raqleJ dur ur 3w188n4g

'pnolJ

_'pnol ,1deay

e l.r[

pn{ puarJ e a>lrf

Mors uMo srr rraual

V

o*o,,,,1X?,'o5#ffi;L:ijil,

lly

(S) au!7 .anle+s

uuraN

.lelrrre rnod Bur,,(yru8eru,oqca fiyerp e q (S) sarrol ,n[ iu17

rrcppqrnodpue,.":,HiH;t:;T;#H,,"j,"i1;l,.lh* 't{4EM plo8

Morros lueJul

1ey e

a1q Buro8

nol ps aaol

Suog Stmrrop

drue;ur premo+ sapn]Fl€ rrar{+.lea^ar o1 .,ira8eurr asn sraleads aql urtorl az[leue dessa pazrue8ro-qazvr e ur,uagl .a4e1g UeIIIIM pue qteld ernldg dq suraod Buuaoloy aqt pear lpyare3 ('arors Aessa Ielol alD Jo prrql-auo se slunoJ uo4sanb srqJ .sa+nunu 7

uoqsan[

@f )

",*

Jaqloru rnod aroru ou ur,J 'sllelvl se dpnrntq punor puets a14 dlaps rno sMopeqs ssaupa>leu rnod ,runasnur

Surdrd,paleu,ssaldpll

J plrom sno.ra8uep aril oluJ _ 11dau raqle; dru,paueol8 raqloru dyq

JeJ

aq1 Buoure sJa>p{C

q+earq-q+otu rnod lq8ru

a{elg ureqlrM_

aql

0'-€u4+ palsa88ng)

Question

3

(Suggested time--40 minutes. This question counts as one-third of the total essay score.)

"When

a

true genius apPears in the world, you may know him by this sign, that the dunces are all in confederacy

against him."

Swift -]onathan "Thoughts on Various Subjects, Moral and Diverting,, In some works of literature, the main character often finds himself or herself in conflict with the social or moral values of his environment. Choose one novel or play of literary merit in which the character is at odds with the people around him or her, or with society at large. Write an essay in which you explain how these conflicts are essential to the overall meaning of the work. You may select a work from the list below, or you may choose to write upon another work of comparable literary merit.

Awakening

Man's Fate

As I Lay Dying

Mnrat/Sade

Catch-22

Medea

Crime and Punishment

Miss Lonelyhearts Moby-Dick Native Son

The

The Duches:s of

Malfi

A Fan's Notes Hqmlet

Nausea

Heart of Darkness

OId Goriot

Hunger

I Knorn Why the Caged Bird Sings The ldiot The Iliad Inaisible Man

One FIew Over the Cuckoo's Nest The Scarlet Letter The Turn of the Screra Under the Volcnno Waitingfor Godot

King Lear

Wuthering Heights

Long Day's Journey Into Night

STOP

END OF SECTION II IF YOU FINISH BEFORE TIME IS CALLED, YOU MAY CHECK YOUR WORK ON THIS SECTION.

I64

I

CRA(KING THT AP ENGLISH TITERATURT EXAM

QUEST|0NS

r_15

il?.t-ffi1f,:TTrlT"i":1"::,'"r" 1t^T!:!:q,:,.1

lli:?Y*'h:#flfi:"::i:":f

:::li5ffi:f

fi those elements on

:ou,"r

by British aurhor John Fowles, is a serecrion

t':*tm;rililq;'q;#rffiff

?;ffi:yi.-';#ffil".'*";

iftie:,:*::y:,1,.,.:l17erements,u,,"in"*"*i'rilH'.lTffiilXHH.Til selection r,,;h;l'..

tlit it every piece. But after you have T^",::::1S: ""lr"lor is imporranr in rhis piece, as T .rn situation ffi'"T"Xl:l1p"ll:,,:.f:,,""*y",i,no,1J;;;br.i;*",.",r'f,fJt"#.."?,i",::ffiJ"?Jil: is reflected in the setting. 1' A You may have read answer choice (A), which is a straightforward answer, and thought, and then puzzledou"r *,.-oii?r choices wondering if you,d missed something' Perhaps (B) and (C) seemed to ue possiut. unr*.r, also. Most of the time a Pronoun refers to the last noun that has been u"t there are-e"..d; to this rule. In this case' the narrator is telling us that phruxos, *hich is in the centei oi th" tu.ar.up" (B), is as memorable as the landslc"p..jr. I,vhen you tootea at choice (c) aga*; you wourd ror rocatioir ra** imagery crJrce ror courd not precede the pronoun' not the other way around. Your only logical choice

"oh' that's the answe1,"

""*J,

p,l

ff;:;::lfrT;"T:S:ilS;J#J:a 2' E Remembtt tlil^I|*

"L"#r?fust

u-t given choices that contain two words, both words must corI.ot rectly answer the question' If oie of the wordr ir then the entire choice must be eliminated' Thd question also "of".."rate, tests your knowleage or hterary terminology. Choice (A) could have been eliminated immediat"iy becaur.iir.""**ror states that it #as ,,simply and effortlessly beautiful'" Thatalone would have eliminated choice (A) because ,,pretentious" indicares i! was showy and not simple, rh. a.r.ription is arso no1 . f;;; of a hy_ perbole. Choice (B) could have been a close choice b";;;1

ptrur" "amethyst;t."t"g;;;;^il.

3' c

udlectl'e;urned;rtt:#;!:::l1ff:?5i:l;rT$;

However' the term "elusii'e" eliminates the answer ul.urr" ,,elusive,, indicates that it is intangible and mysterious' The concrete comparisons that are used in lines 5-10 would also help to eliminate this answer. choice (c) indicate, tr,uiir,u comparisons that are made are farfetched and do not have any concrete references. This is not the case. Choice (D) indicates that there is no depth to the impression thelandr*p";;; made on the narrator. This is the exact opposite of what has occurred. (E) is tn" .orruit ur,r...r. The comparisons used in these lines are predominately similes, and when you .orrtirr.re reading, it is evident that the use of venus and the use of ihe whale ure symboii.. v.""r, the brightest planet in the sky, is used in contrast with a black whale **.ni.,g ,.u.hro, venus is symbolic of passion 1 and romdntic notions, whereas the whale ir u ry*u'oio?.urti,ry and pragmatic ideals.

Make sure you understand what this question is asking. To answer it, you need to first think about what the narrator's tone is as he describes the isiand H.:, its beauty. Now' what is the effect of this awe? Choice p..tty tal redundant'and does not really address how the tone advances his anriciparion of the one "facade?" This choice is only partially correcf the narrator is expressing something about the setting' However,

*.-r somethint.il.;;ii;;;"".. .-pil;;; y; ryu!,*ryi;-.

i."ii;;;y

he is noioffering a logical of the lack of towns on the island (D); this does not really suit the toneif thing with (E)-the narrator is riot offering u objective analysis. so what aou, i,i, u*ed :?Jd., description of the island,s beauty do? rthighlights ihe contrast'befween nature and the intellect, choice (C).

I66

T

CRACKING THE AP ENGTISH TITERATURE IXAM

t9a

-

sN0tlvNvtdxl 0Nv sulllsNv

:l IslI l)ll)vud

-rldarrns ou are araql asneJaq (g) aoroqc uasoqJ aAEq lou ppot{s no1 'uoqsanb srql ol ra.MS -up parro) aql sapr^ord prguor sTqyaJIM slq dq paraprnru aq ol dpo'd1a;es aruoq paurnlar uouurarueSy'JapJnu aql JoJ uouuraure8y uo a8uarrar panrol 'dorl;o ualaH snorueJrn aql ol Jalsls oqe 'e4sauura1d13'aprvr s41 :alq8nep sr-q 1pl ol uoISIJap sn{ pazITeuoI}BJ aH .d1aps aruoq rirn+ar lq31u aq leqt rapro rn spo8 aql o1 erua8rqdl ralq8nep sl{ parlJlrres aq

,arn4redap sr-q ol JorrcJ 'dor1 o1 snassdp6 paruedurocre oqm JoIJJErvt IaaJD e'uouurarue8y 'nod puelsrapun pajse Suraq sr lou lq8rur nod lur{rvr Jo Jo aJIM aql s€rvr e4sauura1d13 uaql dSoloq/ru pue dpa8e4laarD ur areld rrarfi rraoDl lou op nod I'suolsnile are uou -wauie8y pue e4sauuafi3 yo sarueu aql leql IJEJ ar{l q}rrvr nod saprrrord uoqsanb

sql V

'L

'ssa.rdxa

ol Surtrl sl roleJreu aql leqm;o Surpuelsraprm qldap-rn arotu e sapltord 11 'uousanb sryl roJ ra,r,rsup lsaq aql sr (3) acroq3 'qder8ered sTq+ ul pasn Suraq lou $ auol dpeloqx e leqt 'uotssardap luaprla s111 'arualuas lsel aql ur sralsuoru pue sqdudu Jo uoquaru aql qllM saleJrprn spJoM asoql Jo Jarl+laN t;arraa,, pue ,,'aJueJqru3ts,, spJoM aql Jo asn sr-q dq luapl^a sr TIJII,vr'qder8ered sr-qt q apnlple s,JoleJJeu ar{+ }o alnpld alurnrre ue apnord }ou saop (6) atroq3 'salels rolerreu aql suosrredruo) aql Jo auo o1 dluo srayar (g) a)rot{) 'puel$ aq} (V) a)Iorlr aser srql u1 'paprlord seq roql Jo apnirTos aql;o uoudursap aql ol sraJar dpo raprsuoJ -ne aql 1eq1 dra8erur aql pue a)roqJ pJorvt aql lsnur nod'auo1 qqlqelsa ol rapro uI 'uorsrJap B a)eru nod aro;aq qder8ered arqua aql le {ool e ale} lsnur nod'ue8e arug

)'9

']Jarro) sr (3) 'aro;a,raq1 'a8essed aql rr ralel lraga lear8 ol uodn papuedxa q leql uosrreduor e dn sps ll 1lur0 ueql aroru saop qder8ered qq1 1nq'ftneaq s,puel$ aql;o srsdpue Ierqlrr e sraJJo qder8e;ed puo)as aqI 'uoqsanb aq1 srar*rue ro luaualels aq1 salaldruo) lsaq leql a)Ioq) arll lJalas lsnur nod lerll raqruaruay 'a8essed aq11o Surpuelsraprm rnod;o anrl aq deur s1q1 q8noql uala '(q) arror{) r{sllqetsa o1 dlaq +ou saop qder8ered puoras aql 'uolssardq qq;o uoudursap alern) -Jp ue lou sr (g) aJIOtp uI ,,Ssaulseaun,/ pJolvr aqJ ,,'saJosa f,a,, aqol s8uppnq uJaporu aql sJaprsuoJ pue dlneaq aldurrs Jo sruJal rn lI saas aq pupl$ aq] uo saAIrrE aq uarl A'a8essed sr-ql Jo anr1 lou sr ,,alue1dalJs,, pJolvr aqJ 'aleJnJJ€ lou $ (y) actoq3 ,,'uolsnlll,, ue osle ale seq aq duouuuq;o s8urlaal aql leql srneal all 'asrn8srp E seq ter{t 3tm4 dpo ar{+ lou sl sHi 'aloqm € s€ leql surual ralel aH ,,'ape)eJ,, e seq Suppnq aql 1eq1 sazruSoJar lolerr€u aqf, uorlJalas aq1 ol qde,r8ered puocas aql Jo aJuel.rodun aql uo palJar o1 nod slse uoqsanb srql

J'9

'(q) acroqr le pallrre a^eq ppotls nod'gg4 q8norql'uoqJalas sH+ ul sprom aql Io [e;o Stmreaur aql l'rouT ]ou p1p nod;r uarrg 'aders -puEI aql uo uolsnrlrn ue pue rusqenraunuoJ urapou Jo +lnsar aql $ Ialoq s1q1 'ace1d Jo lno dpuapr.ra q l€ql pa[qo a8rel dral e salldun tpsaqo,, proa aqJ'(d sl aJlot{f, Stmneurar aq; .uoueJol IDI,r,r op 01 seq lerp qder8ered snonard aql uI prmoJ Ilelap € sr (g) aotoq] 'aqlrrsap ol pasn Suraq sr ,asaqo, pJon ar{l leqrvr 3w>1se s uoqsanb aql 'uo4sanb aql Ja,lrsue lou saop ll 1nq'saleru roleJreu aql l€ql dSopue aql ol uoslr€druoc e palJar saop (3) aJIotIJ 'uoq -sanb aq1 Jamsu€ lou saop qJIqM'Jaleal aql ol uonelal ul lalor{ aql Jo uolleJol aq} qllm sFap (y) atroq3 .uaddeq lr{3T,o 1r q8norpp 'proivr e Jo uoqugap Tooqlxal E roJ pDlse aq nod p,rn dlarey 'lxaluoJ uI prolv\ aql yo Swrearu aq+;o Surpuels.rapun Jal+aq e ia8 ol sreadde prorvr aql

JaUE aJualuas auo pue sreadde

prom

aq+ aloJaq aJualuas auo lseal

lu pea5 'pasn sI

pJorvr

aql ,.1 oq aas ol IJaqJ lsmu nod taqrunu auq e ualr8 are nod;r'p8ro; l,uocl 'azls e sa]eJlpul ramsrre aq+ asneraq (g) asooqr dlsnoauorra lqgr-w nod uoqrugap aql uo dlar nod;1 'sreadde proa aq+ q)Itl1v\ uI aql aq+ ol ratar ol atuq aql DIel nod leql sallnbar uoqsanb qq1 ,,asaqo,,

c'v

tious events that follow in this passage, as is evident by the narrator's understanding of the misconception of the harmony he felt. If you chose (C) you may have recognized the names from Greek mythology, but there is no mention of the history of the island by the narrator. If you chose (D) you may have recognized the Greek names in relationship with a kingdom, but the word "grandeur" is not the word that would be used to describe the island. IIt is its natural beauty that captivates the narrator. You could eliminate choice (E) because nowhere in the passage is there a reference to violence. Once again, you might have been able to make the correct choice using the Process of Elimination.

I58

I

8.D

This question requires you to have an understanding of the vocabulary used in the choices. Once again, you must remember that both words must accurately describe the speaker's impression of education on the island. ln this case, choice (A) can be eliminated because the education is a solid and practical British education, not superficial. You can also eliminate choice (B) because it may be annoying to the narrator, but it is not archaic, merely adequate and current. Choice (C) does not provide you with any choices that are accurate. Choice p; may provide you with the idea that the education one receives there is inconsequential, but it is not perplexing to the speaker. He understands that the education is adequaie, but he is frustrated that it does not involve more of a romantic notion of education. Therefore, the best answer is choice (D).

9.A

The speaker finds the school constricting and the students exasperating. Th.y preferred to talk about cars rather than poetry. He found this preference of theirs frustrating, so choice (A) is correct. If you had chosen (E), you chose an answer that was close to coriect, but it was more than disappointment that the narrator felf it was frustratiory as shown by the way he developed his examples in a repetitive, almost sarcastic manner.

10.

A

11.

C

This type of question posses the most difficulty for students. Once again,you need to look carefully at the question and the choices you are given. Eliminate the choices that do not supply the correct implications. It is hue that the narrator believes that there are shortcomings in the British educational system. The education fostered an interest for science and little for literature. If you decided that choice I was a correct answer, you are on the right track. You can eliminate choice (C) and choice (D). if you look at choice II you will seJthat the "myopic" (limited) vision of the students who only want to learn scientific information and not that of [terafure, also makes selection II a valid statement. Because the choices that are left all include If you have to look at III and IV to determine whether or not they also apPly- Selection III may be true, but you need to refer to line 80 to make sure you understand what the object of reference is to "mole-like blindness." You can easily determine that this reference loosely applies to education. In that case, you can eliminate selection III because it refers to all inhabitants of the island. We do not know if this statement is true based on this selection. Now you can also eliminate choices (E) and (B). Choice IV is not true, and therefore you are left with choice (A). This question requires you to know some basic literary terminology. Parody is not evident in this passage; you can eliminate choice (A). Aposhophe, which is often used in poetry, is not used in this passage. You can eliminate choice (B). Repetition is often used by good writers, but choice (D) indicates that the attitude of the speaker is emphasized by theiiusage. So choice (D) is not the best answer for this question. Take a look at (E). Hyperbole is not used. Now, you need to decide between choice (C) and choice (D). This passage,like most literary passages/ does depend on the author's use of imagery to make his point. So keep choice (C). Thai leaves (C) as the correct answer.

CRACKIIIG THE AP ENGTISH TITERATURE EXAII

691

I

SN0lrvNVtdXt 0NV SUt/r4sNV:l

lsll

l)tI)Vud

(g) lnq ile /aseJ sl{} uI 'uoltJalas aql yo Surueau aq} a)ro;urar o1 dlaq sluaurala leqrt Suqse sr 1t-a8essud aql ur pasn alp sluauala dreralqleqrvr nod Surrse lo.r rr,rorl -sanb aq1 'uo4sanb arfi ]e rool o1 s nod Jo pa{se Brnaq sr 1eq^ Burpuelsrapun o1 ,{a>1 aq1 'suoqsanb asarD tn pasn uaag aleq l€ql surral dreralTl arn IDIM JerTrueJ 1o,., .rn nod;r looq aq] Jo )Jeq aql ol raJau dSolouru.ral ;o e3pa1mou4 rno,{ uo pazzmb are nod ,ure$e arug

!I

'9I

Ja SUe pAJJOJ aql q (v) 'lQnop e lnorDIM 'alelrdordde '1r rage sauor +eqm pue auII aq+ arotaq 1 1sn[sauror +eqM lP {ool lsnur nod 'uo4sanb aruara;a; e ur 'raqruaulay dlsnor,rard sr{es ra>1eads arp }er{,1a lou sI leqJ'sllH aq+ uI aruq srq,{o[ua ]ou IIIzvr raleads aql lErD salerrpur (g) acroq3 .parroln q sIil 'sarnllnJ oml aql uaamlaq dltsotutue 8ur1aa; e saleJrpur (q) acroq3 .1.rJur"1e1, ;o "ql o1 azuodsar ruoDrJJns e iou pue ;.1}eqr pale)ry,' relap e q srr{J 'raarD rplridr.n ^.."4 dlsnor,rard rolerreu aq] pue a8en8uel >laarD aqr qlr^ sleap asneJoq (3) alernunla oqe +r ueJ no^ 'anrl lou sl leqJ 'uolsnlll ue se1\{ alerurlJ arp leql sale)rpur aJror{J aqi asneraq (g) .pqur pue dpoq alernrurTa uer no^ '(v) sl uo_4sanb ar{l roJ raMsue tsasolJ ar{J uaa,ropq duour -JPr{ Jo sSurtaal s,raleads aq} ol sJaJaJ up se,vr :auq rrunr"jn, .ql ,,uorsnllr }L,

V

''I

'lrarrof,

9re

i.8roj

1,,.roq

ramsue lsaq aql s (6) arroq3'(g) aleuruqa u€t no1 dem ieql IaaJ rulr{ a{eru lou plp slFi aql'sauq1e luarledur i1a; arrer1.,{eru raleads aql q8noqqv'llaa se 13j a1lulu11a .}r ueJ no^ '11a1 ra>1eads aql leq,/vl1o alsoddo aw sl .an4 aJror{J a}€unurla uEr no^ o) lou '{ea^r q sHI IaaJ turq aperu ,{aql salerpur (g) arroq3 'aJnleu rgr,r snoruorureq IaaJ ur1{ apeur daqg '1aa; raleads aql ap€tu sruq aqi noor{ ratur o} parse .rn no1 dlinlarec l'**rl"l, aql pPaJ ol paau nod tanarroq 1uo4m1as slql uo paseq sJaqlo qlr,t,r alelnmruruoJ lou p1p ra>1eads aqi leql ra;ur dlqrssod ppor no1'alrleJrununuo)un suearu ,,,Jrrro)eL,,(y) acroq3

'(g) sr JaMSue aq1 ,aro;a;aql .(d aJroqJ se asuodsa.r orues aql saqdurr (g) arroq3'paprlord sralseu aqi ler{+ uoqerurolur Jur}uans due parno1ap s}uapn}s aql leql salels rolerreu uaqm parldtul q (d a)roq) .luaualels anrl e $ arror{J srql .s3u, 9yl -punoJrns rlaql alelf,ardde 1ou op sluapnls pue sralseru leql salerrpur (3) arroq3 .sub]lsanb raqlo uI aleJn)JP se paztuSocar dpea.lp arreq nor{ leql stuarua}pls saleJrpur saJror{f, Jar{+o aql lE Tool V 'anJi aq iorruE) 'aJoJaJaql '1uaura1e1s srqJ 'uoqeJnpa qsrlrJg e Sulr,rarar pup {aaJD osle aJe IootlJs aq] uI sluapnls a{l 1nq's8utpunoJJns Jlaql ol uoquaile ,{ed 1ou deur s1aar3 aqJ '(g) Ioor{rs ur sluapnts aql Jo asoql ol }ser}uor uI lou $ puelsr aq} uo s{aarD aqt Jo apnlqle aql +eql 1vlou{ no^ 'pue18ug uI q ll se arues aq} sI soxenld uo loor{)s aql lo araqds -orule aql.an4 sr (v) aJrorlr rn luarualpls aql ]pql MouT op no^ .lrrral aql puelsraprm l,uop nod;r ua,ra 'anl1 are sluaruarers aw JI aas pue {ool p a{eJ 'uoqsanb taaf,xg .rn r, ,rqi l"qi Jaqruarual lng '(salrural Jo isau e) ,,friuywta|;o Surueau aql Mou)I l,uop 1,, des dew no1

(I'€I

g

.zr

QUEST|0NS

t5_27

The passage is by Christina Rossetti (78g0-g4),and

H,=:ry;|uTt#',ffi1,H:;ilf'*"

was written. when she was in her early thirties. is tvpicat or nossetu, who was f.,.i*i,n iil-hearth her

The Rossettis' christiria and her brothers, william Michael,and of an influential mid-nin"ttt"it'-tt"iuly Gabriel, urt, movement catteatrre pre-Raphaelite were at the center Brotherhood. pre-

?*r.

*?iJ;.','i:.:tTits.T.,Hq;lm,:.#f r***x'_1;T;,,fr ff::filce(o*en,^s.d Chrisrina's brother Dante (;g;"bry in. tu#"r;il;;; iiarhaerit the truly cheeseball uttt movement) is gu'ry of one or ttur.ir"rirrl'"t of 1**q r1i* ;*uJ bante cruri.iii"rrJtti , *ir. died, the

FTi":L:i.T:'.',1J1'#ryU:"*mt!:ii1f""".il'Jthecasket;i,il;;Ah,,oves.;; up

his poems back. The,rar,

r*gh,

n"ffi;

is on Danre, whose r,rl*".tl"t^1111,d"q

so he

:glriJ g;;

;,ff.,il:,i:ff fi :o1T".d;;s;"".r"g,",p".tio;"ilJ1[J#ffif:n

j:ilu"r*HissiJter

li:T_r"ffi :lfrli:!:iF:T:?;tn*",:'m:*iti*iTi'"#:Effi

;fii,.,,...*.i.,

any years spent The poem on the test (like almost ev.eryfhing Christina Rossetti wrote) is a meditation on the

Although the bulk of the poem;t'-"ttg"s the test lav several trqys jor ti.;;;"ty. making assumptions tirat can't

i, u.."rribi" tl -ort

r.rdi"g;;;

readers, the questions asked

on iit.rprgtir.,gpoety,ui o' guura againsr

b.l,",ii.a.16 incoirect .riii.., ,r..r, sugges*he FF;;:#;;r';:". on her a**'ti.J. ri"' shourd ;t;;;;;; that the p";,;;;;;i-,. 5iil:ff #[:H,T .o,,t._prutio,, Ano,her**i#,i5,i?f :;f.T::*:ifr ;ifffi **:*;[1{:xru;::,.,*,,f l;".,. tions ask about some or tnt pou*aJ;;;; p"iitr. Thu.. ,* ,.**r questions, for exampre, about the

;ti"'i::T:l]'tT;1,:::.1'!',T.1?$1:;:*:t*###;:poem,sri,,ui,tu-u

co-pri.uuo.,,

This long-standing tradition i""r[^irg with thi spidtuar forces of hopelessly old-fashioneddevice' "r the cosmos may seem a o* up to the p."ri^iJuy continue important works using this to create interesting and tonutn,ioi."it e nioss"tti logue with the metapliysitut not gnry has rhe speaker in dia_ matterJa degree further in the soul speak with ihe the iecond stanza byhaving voice or tn"pu,i."ioilowingihe raTil""rt.n what the past doth witness say:" the soul presents what and trt. pJri n"rio ,uy ibout human mortality. you needed to that in this sranza thlnlst understand is nor teing a1r..ttyp.u*"r.J'rr r ,p*5*. not even being quoted;the is probabry soul is inte?preiing'ti" uil.n, of"rithe principar narrator. This a tangled piece of rhetorical .ot',rt

** *o'td,l;;'k*

r

6.d;"ve1,

purl;;il"

f";;il;r

u.roi'rrid .u.rr", ,.,oriri.r"i.r.,ts overall, the passage, ,11.1 ,og"rhJ*iin its queJons-" ,ii1. some problems. difficuliend or ir,u spectrum work you wi' see on the Ap Engiish Literature and Composition of Exam. 16' D As noted in the generalnotes to tn. this, is ajgugh question. answer choice (E)' five' But Most students choose lu:11r.,the past is notl. speaker. Thelast is being principal narrator by the sour. interpreted for the d"otrr*:ho,:^.. rt'rir"prriricated read-er, ,"fuiir"., pi.t is (A), one. The reasoning behind cnoosing ral ir rr,ririilt* o.nly the poet is speaking; the soul, worrd, and^cojt"p*r""t jrra .r..r.,it, the poet. In this ieading, the poem is a kindof int ntir-o;;i"g* thepoet sorts out her feelings about death and the afterlife' ntit ryhi+ i"i.rf*,r,ron Tis absoru*iyi,i*rrule (Rossetti intend for you to think sh" certjdy did not hua alturily r,"ra with the world o, The problem is that it is coa;. an iitrrprttrtiir. rrr. " ,,How '"i*,does many speakers the

il;'il;h,"

.;;:J.t"" q".riilffi,

I7O

'

CRACKING THE AP ENGI.ISH TITERATURE IXAM

lll I

SN0llVNVldXl 0NV SUITYISNV:t

lStI llll)VUd

'qluvr pauJa)uoJ sr'aloqn e se uaod aql Jou ,uoqsanb ul aql aql Jaqlrau leqt sauaxue pnl1r1ds ro'luasard $ auou ararl,l- ssaq11 Suaas raqlra's8urpeaJsnu snolJel raJJo saJroqJ paJJoJt4 aq1 'ssaro,rd 3ur3e prnleu arp to uoq -dursap aroru auo 1ad sr a8eun aq1'dlucrroqdelaur Xlpr3p ol luearu sI,,derre au.RD 14 tl+orx e,,leqm pa{se alalr,r nod'dlprrseg'alqno4 qJnru ool nod uarrr8 aleq l,uppoqs uo4sanb aW

iI

,TZ

'(g)'ramsue paJJoJ aql qlyvt dluo ga1'a1qno4 tlJmu ool lI ssor)'Suorrvr ra.&\$re aql sa{eu lnoqluvt Jlasmod pulJ plnoqs nod dem srql8uT).Io ,{ple1rar s derap prrsdq4 'parroJ 'rurat sI JeaJ e 3uo4s ool qJnru sr-ql ^'JJo 1nq'paleldrualuoc 'lJaJJoJ dlpuuelsqns sI € Pzu€ls -ryq dpo s;.Zezrels ro; ,,derap pcrsdqd Io rea!,, 'larra,l-o11 JoJ ,,aJIIJaUe aql Jo aJu€ldacoe aurocla./vr,, pu€ 'ldacle ol q8noua asolD sI ,,pqqaq Ual aq lsnru ler{l plJo.rv\ dlqpea aql roJ e€1e1sou,, '1ezrte1s JoJ lel11 uosear plnotls nod'aldurexa ro;'(g) r4 'saJroqJ Jarrsue aql yo Sqprom aql ol uonuape asolc ded'suo4sanb Jo sprryl asaql q 'Suorrvr raasue wq '&uotm r sa1€ru U$s q parJo) sezupls aarql Jo lno oml lo Stmreaur aq+ Eag l€rll '02 leq/vr ro; dlryarer aJrorlJ ra/rslre qrea Surpear'gg4 asn ol se/vr da1 aql uo4sanb sgl uI iI

Sqrt1 uo Jou suospas arp uo raqllau saerp 1eq1 ruaod eq1 'suos€as Sur8ueqr aql Jo pu€ ;o a8eur auo aql s ,,p1o8 dW.rl lsn5,, Jo aSerur aql'€I aql r{ ,que1d dlprcadsa's8tqq1 Srql[ uo s/{eJp 1eq+,{.la8eurl Jo asn a>leru ile sJalvrsu€ FarJoJlrI aqJ, 's8tr.n1t

.uo4daouoc

J '6r

ruo{ sn qql,t- qleap Jo paas aql .,{rrer

arvr toqdelatu 1ue1d raqloue asn o} aql tn asuas qleap ro lqprq tuo4 luaurunm sr teql ,,uaTJI4s looJ,, a.re praua8 rn suerung pue roqlne ar{I'a}H uEumq uI Uleap 1o aruasard FluaueptmJ aql q ol sraJar uolll4s-loor leq A 'p dgleap sI Jolprreu pdour.rd aqt luql 3q1ecpu1 se ,,ua{Jlrls looJ,, uo dn lcrd dpuanb -ary orlm ,Euapn+s dueru ro1 auq la{Erualqnor} P $ I atnl 'aJII ueunq;o lftopafe4 reaupl aw r{llm suoseas aql lo ssarSord prmdr aql lse4uoJ ,,'f,eJap dq1 plpqar lou lpt{s 'ua>Pr4s loor 'noq; /:f,eyt1ul pnq pue Suudg u;lasdur aqtolr lleqs L, 'araq uoqsanb ur saull aql

.lamel qlrm salenper8 SrmrmorJ Jo uoqrpe4 dltsralnm Ielalparu aql

fl '8I

ruo{

palrJap rural p 'aryatnalettuq.rnod pauJea aleq nod leq ueau pm aa#ap s,JolaqJeq e qllzvr a8allor ruorJ uorlunper8 's.rouoq yo Stpleadg 'apatnal4aod sr al+1} 4aql laod puorluu aql se parouorl are aldoad ueqrvr 'depo+ ua^g d4eod Jo poD uo4ed 'o11ody Jo ramolJ cqoqtuds aq+ servr (1arne1yo dlaueur e sr deq) pmq l€ql sr slaod suoquaru dlecurcads ramsue arp uoseal aq1 .a1ed ppq s,Jesa€3 sngn{ 8uuer1 aas sdemp nod q}earrur leql q se 'qlearm Iarnel e sem

aql Jo ,,ppaur p1o3,, pqrSlro aql 'dAaod Io le^t'qrods u qdumr4 yo uoqruSorar ur papJe/vre s€,rt salpal r{eq pue IaJnEI Jo puepe8 e '&anos ueruo1 pue {aarD }uallue uI ra+PI aql yo aldurexa ue s1 uoqsanb sr-q1 'aznrSoJar plnoqs Fnpl^pul ppar{lom e saJuaraJal luJuolsrq r{.rera11 asoql pu€ ruslJ +noqe {se r{lpuorsecro daqt pue'('ap'1a1dnol'lauuos'.roqdepru'apuns ''a'D urroJ padxa padxa nod uer daql daql'mool nod ol /vroul -qrro d.re;a1q 1o dSolouurJal Jrseq aql 'arnprd nod aql qloq'pear ra8rel aql pup spelap slaa1 SII r{)F{ speJ aruos ar€ araql lng

rila*Xp

^

Ierralelu aql puelsrapm o1fiIIIqe mod lnoqe sr lsal aql;o luacrad &aqru o1fiq3rg'1,uop nod ro lI /roDI raqtra no1'lsal aql uo suoqsanb a8palmoq arer dlalqqar aql Jo auo sr slr{l 'sraleads rno; sluasa.rd

11

'1sa8 .quasard ruaod aql leq/vr JoJ d1du4s lnq uoqelardralur ue roJ 8uryse lou sI uoqsanb aq1 -8ns lq8nu ruaod aql leqm 1ou 's1uasa.rd uraod aq1 lpt{/vr uo s srsegdura aqJ ,,iluasard ruaod

J

'tT

c

22'

The line runs on from 7 to 8. This is another terminology question. If it gave you any trouble you should refer to our section on literary terms for tdAF Composito get rid oi tio* u"'*.rs vou are sure are wrons

E"J;lil?"#;nd

ff1ff:il*ii";Til:,Tff:,1:ffii3i 23'

B

This is a question that many students get wrong. Always retum to the passage. The third stanza presents a dramatic reversal and direction;t;ufig"ru.,g imag-*riirg ery from the previous stanzas with an antithetical meanin!. In the first two stanzas,spring and all the image-ry of spring are used to.tup..r".,iilrh; energy, and life. you might easily think then that-winteq, ui spring'r oppo.site, rwr.ru't, 1rj agi'g and ross of vigor, or per_ haps (c) the coldness of the g.ui., that is, death itself.

l h.,.ry*,

of winter nthe third stanza'in this stanzaGod says delay'" \{hat follows are images of spring ttow

il

tli. q"";;;k" i"i,n. meaning tharlow 'winter passeth after the long

clJu.tf ,iJ to death anh ur spring in the final stanza is a metaphor for the jJy of ,e.r.,ioi *irn coa. In the final "it*rir.. stanza, God offers death as a joyous springlike o..urior,. It is earthly life, separate from the Maker, which is the long Winter. 24.

C

25.

e

26'

c

27

'

As with all questions with longer answers, you must read carefully and eliminate when an answer is partially correct. Partially correct means all wrong. otherwise, the reasoning behind this question is fully covered in the

expla;;;;;stion

23.

fnferstanding the lines in question is not as much about the lines themselves as it is about letting them make sense in the overall context of the poem. If you understood the bulk of the poem, then this question shouldn't have been difficult. If the poem itself gave you trouble' this question might have as well. The incorrect .h;i;.r offei various m#readings and over interpretations. one of the easiest questions on the test. This is essentially a vocabulary question, but chances are you were unfamiliar withthe pusugJr ffi of the word ,,sptay.,,Figure out the meaning from the context. None of the incoiect ans'wers makes ,.rrru ir,'.or,text except possibly (A), and we hope that befween (A) and (C), you chose (C).

D You are certain to see a question (or two or three) like this one

on your test. If you got this question wron8, brush up on your skills with our section on gramrnar for the Ap English Literature and Composition Exam (page 53). As outlined in tiat section, th;;";;way to figure out the construction of the kind-of sentence ETS likes to ask about is to rewrite the sentence (in your mind-you shouldn't need to actually write it down) into a more nafural form' The sentences ETS ihooses are never straightfor#ard "subject, u.ru, air*t-object, indirect object" sentences like 'Jack threw the ballto me." The sentence that begins on line 24 "Arise, come away, night is past and lo it is auy, rrrry rorre, My sister, My spouse, thou shalt hear me say," should 6. ,.*ritt rl

"Thou shalt hear me say, 'Arise, come away, night is past and lo it is day,

My love, My

sisteq,

My spouse.,,,

Notice we've put quotation marks

urgTl-*hut

sentence would normally be punctuated. to see that ',ThotJ,,is the subject.

II2 }

CRACKING THE AP ENGI.ISH TITERATURE EXAM

God reports he will say. This is how the If you rewrite^it in this _u*6r, yo, ,f.,""fa be able

I

tll I

SN0llvNVtdXl 0NV SUl,!ISNV

:t lSlI

l)tDVUd

peal a^eq deu oqzvr luaprys aJeJ arD q4eJ o+ pauSrsap auo ,sra.,u.sue de4 1e Jo IIAa lsour aql dlqeqord s1(q) a4oq3^.p*qr'q"r,dFyr^D .s{A[,uoreapqrre aq] +ou,arpnord .rCI ol sraJar ,,'palraduaq sl doqslq aqt ,1oe; q,,,tqde.{etnd"W or.r, .rJurr,rn4 ayJ .a3en3uel al4darap Jo asn raqloue s (g) arroq3 'o;ra,r s,uerud8rarr e se aro'nq de4 e s (y)

bj o*grdl1pryr" u;iu^

,,,4tretr spJolvr aql saas or{^, Japeal ssaraJeJ arr+ aleus o1 pauSrsap ,Ja^sue arroq3 TEarJ uoquarur s,Joqlne aql sa{e,' .'JIAJ ,qde.r8e.red os ,,'orpnoJ4

Jaq Jo saSap,tud

II'J aql,,

1op,, luanbasqns aq+ olur uo4rsue4 aql pu' 'linrl.ro, uI pinf irn sanur^ s,rliue.rp .srl .ise4 +no J -uoc dq arpnord .srl{ aqrrJsa_p o+ pasn s1 ,a;yvr ,,.ror""pqrre aq1 dpue.lD .s{Ai yo aldtuexa aq+'rape.reqJ s,arpnord's.r1,1 Surqursap ol pa+olap sr r{Jrq,r,r,r8nrrnd rqllo aql uI

ffi"r

.a8essed

arfi ul plalJe raq+rnJ uo.r; a8en8uel aarldarap alel (f,) pue (v) saJlot{) 'a8er'rreru srl lnoq' servr a8essed aql ieqi drouraru an8e,r aql ro ,,rr}sourop,, saserqd aq1;o Surpear ssararpr uo paseq a.re (g) pue (g) arroq] .+r apar o1 rddeq se^' rnq 'aJIM SIq o1 ra^od arp paraJJo ,s.ra11eu

^

a^eq +ou

pFolr

aq

J'sauop ur leqi

salu1s aserqd

aq1

J

.20

c

.i€

Surql aq+ dq apeu asrou aq+ a{r' punos dleer 1,uec

ll os,qra,r ro unou e rou s,lr,:iflT;5i; plmos saop ,,Jeln+I7,, pue'ptmos spro./v\ qll,v' op o1 8r4q1ruos seq nrrodoln-ouo +Eq} 3o{ s'/vroDl orl^ luapn+s e ol padde lq8pu (r) a4oqr .spiorur,rirunl uala rouvr 1nq €urql aiues aqr rpnru op (d pue (g) sarror{J 'uoqera88ex. .tn lorr- sr (y) pue ,uor1era83exa ,' al0qrad -dg'puels.raprm l,uom r*r"1r{rnr"i[ t{ilrn rer1*ueJ xpjnrrr.pr r,uare o.{M s}uapn}s }pr{+ prozvr 8rq e ueql .raqlo 'lr 'ro; 3uro3 qJnru a^eq l,usaop (y)er10q3 .arueu uI q lr l€q^,r Jo airsoddo aql $ uoqenrs aqilpueqsnq rarl ra^o sprol oqrvr'd11en4ce ur ,arpno.rd .srl $ i ]r 'qldap lea'r' ur sureldxa a'essed rq rn 1nq ,arpnor4 :rryrtl prol aq1 ,aureu rn ,sr arpno;cl rcr ;o

fl

.0g

1ou'p;qrlezvr e o+ uolsnlt' pc€o1oq/ur e sosn (s) arroq3 'b) se ;o aldurexa ue palurod sE +ou sl +l lnq'o1 lnuqns o1 yooi ,rq rapun asoqr slradxa arpnord 'sr;41 Surqlaruos o+ sraJar (q) arroq3:.rp'o4 .s;61 lou dp*r3 .rr4 ol sraJar (g) arroq3 'doqqq p aJrM aq1;o a.raqds'ln*ron."w Jo p*+*" i"qi^",r pue suoqrqwe s,arpnord ny"xri 'srl J ol dlasop aroru qrnu $aJal €rrudtual afqrw,lyy ,i1op .ruq pa^o) seq ror^eqaq rrlodsap s,alpnord 'srl lvroq aq+ trl os saop lr 'arpno.r4 rc saqrrrsap q8noqlly J Jo lxa+uoJ

3

.67

J1iJjt##ffi,1:rtll;

11

aqi sl (v) os 'pa1e1s seq aq leq,, Jo alrsoddo aql uearu.+,usaop roqine aql lnq U,'.,oI oqe'urser.res aruos

pal'p

nJr"ri#rf::

1il acroq3 .buruparu papua+rn si Joqrne aql yo alrsoddo aq1 iou sr dprelrar rr +nq'nloqrndrq'il p*r1i.ro, aq lq8p ?o ro Jluorr sl sf{+ J} llguI ol raq "ri".D r* +noqe pres q8noua.l,usr aJaql }nq ,aJIM ,,rirnrpqrrn 'dpuerS 'srl{ Jo uorl,rsodelxnf n,{+ rl "qq 13iacroq]-."raq drroriou sr araqr ,aq o+ p*s sr arpnord 'sqAJ leqll dlasDa.rd sr Buuaaunuop rr'nrrg:n;r^ g*r"n**op e Bulqpcsap a8en8uel arrrl -eJn8rl luo{ pa^rrap ursrrcrnbollor n ,, (g) nrlo,{r '}eql lsnt Bulop sqder3e;ed pra,ras spuads uaql lnq ,,'arpnoJ4 srry lsure8e pJoM e aql'alq ,sa1e1s o1 ,rorirrrlui dru jou sr Jo+EJJeu aql 11,, +q8l.u auo lasop sr

V

.gZ

'ppuasso st Burpear asol3 .uJaJsrp ol dsea sde.r,rp l,usr auoJ 'xeluds palsrlvrl Jo sruJoJ Jarfio pue ruotln8"., ard4rmu qrru. ,po1n101uoJ +Err\,raruos aq ueJ saJualuas aq1 tsal aqr uo aas o11de a.re nod asord uerrolrl^rqi ,1rr ,.ra,r,aa,lo11:1sed luarar aql ueql dep aq1;o scrdol aqi qrlvr paruaf,uof, arour su^ ,qazrou ueuofrql duno, .1urrr, ,pue lgglui ua$lr^ se^ lI 'sla^ou arqs+asreg srq puo)as at41,uamo1 ra$aqrnglalors Jo

,ado110.r1duoq1"#;t;irs"rrna ,r,.. 0t-82 sN0trstn0

this novel or its

sequels, in particulTr,Fyml Parsonage,in which the rivalry of l\u[rs. Grantly .w and Mrs' Proudie is given substantial attention. It cert"ainly is not the authoi,s intention to suggest a rivalry,.although he.may have intended to foresiradow it. Choice (E) has some merit' From the description of Mrs. Grantly, it certainly seems as if the author favors women who exert their powerdomestically and piivately. The passage states, ,,before the world she is a pattern of obedience; her voice is never ltud ... she know what should be the limits of a woman's rule." Nevertheless, the ranguage in the answer choice, ,,assert why women should be seen, and not heard,'- suggests tlatine author provides evidence for a position stronger than the one he actually takes.

33'

D Pity, answer choice (A), is best used to describe how ihe author feels toward Dr. proudie,

"her poor husband.'Although the narrator-may feign an appearan ce of objectiaity, answer choice (B), his opening comments make it clearihat"what hl'presents is hiJ subjective opinion' Given that, answer choice (C), emotional judgment,mightte tempting, urrif,ir language is shong enough to justify (D), sardonic condemnition He ii certainly moJti"t nn proudie, and his judgment of her does condemn her behavior. It is choice --ae! is t|o extreme for \-/ tnat -- pj the Passage

,

34'

B

We get no sense of Dr. Proudie's devotion to his wife or of his moral compass, no matter what we might want to in{er from knowing his profession, so answer choice (A) is out. Answer choice (B) is supported by the text.of Ihe third paragraph. Choice (C) and (D) suggest a happy and loving mariage, not the picture painted ly?rrir purugrupl,. (Ef -igt , describe Mrs. Proudie's relationship to her husband, but not the reverse.

35'

C

He is described as "aware that submission produces the nearest approach to peace which his own house can ever attain." Choices (A) refers most nearly to a quality be'siattributed to Mrs' Proudie' Choices (B) and (E) are.not gupported by the text. Choice (D) is a trap answer for those who read quickly and saw that the pasrug. liuu, about the clergy and religious

matters. 36,

A The maid in question has been un{aithful.to her duty. As is par for the course, on a single

phrase or word questiory primary dictionary d"fir,itiorr,'.hoice (D) is offered as an anJhe swer choice, as is a word it kind of sounds like, choice (B). The other choices have no merit whatsoever.

37.

C

The repetition of the phrase "woe betide" accenfuates the seriousness of the servants, situation' It neither slows down the prose, as in (A), nor does it satirize or mock the servants, fate, as in (B)' The phrase is consistent with the narrator's attitude throughout the rest of the passage/ so (D) is incorrect. Choice (E) is too exheme.

38.

B

Even if you weren't familiar with the Victorian use of "chatactet,, as shorthand for ,,character reference youcould derive the meaning from the context of the passage-the maid has been dismissed, and because of this "character," she is unable to find dec.rit u-ptof-"r,t. Choices (A), (C), and-(E) all prey on a reader's familiarity with the dictionary dei#tio.,s of the word, as opposed to theiontextual meaning. Choice (D) is a trap for the careless reader who sees "characte{' and "foot" near each.oihei in the passage and overinterprets-perhaps thinking that the footman is sent to escort the housemuia rroil the premises.

i'

174

J

CRACKING THE AP ENGttSH UTERATURE rxAiJt

sll I

sN0rIvNvtdxl 0NV suti4sNv

:t Isll t)tI)vud

raq ol sarTdde rvtel aq+ slaal arls rvroq lnq'(1arvr se aldoad raqlo Jo 1o1e dlqeqord pue) lsquap aql ol ropadns Jlasrar{ sraprsuoJ dprepar rarporu aql 'aleunuqa o1 q3no1 ool uaaq alerl l,uppor{s (g) 'slsquap al}l[ ddaaro a)U ],usaop lsnf aqg 'u4q lsuleSu pasn)re se snlels s,ls4 -uap aql ploq ar{s saop }JeJ ur Jou 'deuvr sryl uoqenJls drnf drarra aJeJ plnom aqs ]eq] awns -se ],ueJ a,r,r spr€pue1s p8al uErD raq+er umo Jar{ o1 Surprorre lsquap aq1 sa8pnf rar{lol l arp asneJaq lsnf'(f,) ur'dpepturg '(g) aleuruqg 'aremeun ,,d1ap1duror,, raq a{eru l,usaop leql inq'raq uo pasodrur saqnp leJrqla arp Jo atuos Jo aJEMErm aq deur aqs 'saop aqs asJnoJ JO ilrrpra^ e ra^rlap o1 uodn pallP) aq ilyvr ar{s l€r{l ,/vtoDl lou oqs saoc 'dlera1q luarua}els aql a{eJ ,,ealemetm dlapydruoJ,, ar{s sem'(g) uI d1cFl 1rq a{il{ e aJalvr sJaasup paJJoJw aaJq+ Jaqlo ar{J 's,Jaqloru aql Jo }eq} ol alrsoddo dlaialduror apnlq}€ ue saqlrrsap }I 'uoqernu4la lsrry dsua up uaaq a^eq ppoqs (y) arroq3 'sarror{J aqi yo Surp,rom aql ol uoquage plrls ded ol papaau notr dgryareJ saJror{J ra^rsue aql lardralul o1 nod roJ palleJ uo4sanb s1q1

A'W

'srualqold rnod yo (1p 'pe1s poo8 e ol JJo aq plnom nod uraod aql uro{ ro) lsotu pa^los a^eq ppoqs,,{1ryarer [O.I8qsn q)nur s1t{l lo8 nod JI 'Jlasll lerJl aqi ol se Surpear s,aqs )ooq aql ol uoquaDe r{)mu sE sded os pue'urrq uo sada sdel aqs ]uaruoru aql tuo4 dlpn8 tsquap aq+ spug dldurs aqs lsarlnp raq ol qreordde alqero -uoq dpeprqred 1ou pue a8ue4s e sa{e} raq}oru aql 'lsquap e Jo IeI4 aql te drn[aqt uo salras or{lvr (aldoad puoqJrJ pue lerrl leuorlJrl dlarqua ue alprreu lq8nu uraod aql'ralpour s,razr) lnoq€ peJ uI leql arunsard l,upFor{s a,vr) .raqloyr[ palleJ JapereqJ E saqrrJsap uaod aql'dgeuuassg 'pqtu uI Eapr ureru s,ruaod aql lda{ pue uaod aq} Jo lualuor praua8 aq} Jo asuas Sulrorvr e peq nod se 3uo1 se srualqord dueru oo1 nod pasner aleq +,uppoqs suoqsanb aq1 'gura,rg razry udlore3 laod ueJrJaruv drerodrualuor dq ,,s,ropoq Jo aurT 3uo1y,, uraod aq] sr a8essed aq; sr uraod aql

99-rr sNorlsrn0 'uoqeunulla JoJ aleppupr dsea ue aq ppor{s (g) aoroqc 'q8noql 'surer8rda pu€ ssauasral q}oq 3u1te1 '8rn -3ua1pqr aJoru apuru aq ueJ uorlsanb e sderu arll Jo auo ale sa)rorlJ JaMsue aso13 ';lasrnod IrrT l,uop 'salroql ramsue aql qSnorql ssed 1srry rnod uo ll lda{ uala ro '(3) asoqr nod;1 'a{€ru a8essed prrroqdulau e +ou op sroqdepru rvray e 'se1y'arrqdursap dprepar sr a8essed aq1 lra8uo4s lno suels (3) arroq3 '(g) aleunuqa uer nod 'azrrsnga Jo uoqrurJap ar{l lvrou{ nod se 3uo1se os'puoqorua dp.req sr lr'anqrafqns dpregar sr a8essed aql q8noqqy'cr1 -uepad Jou ssalJorunq raqlrau sr a8essed aql-slrmoJ qloq uo 3uo.r,u. sr (y) arroq3 'alaq dep aql al€s IIIM god anrssarSSu pue'dro8alec Suorm-;pq'lq8p-geq aql olul IIeJ sraMsue raqlo ar{l Jo }sontr dsrrcoddq raq Sursodxa dq uaql 'luaplrnba Ielros e ol lse4uoJ ul IsJIJ'smelJ Jaq lno Suqurod dq dpa,l,ap raq Suqrour 'dervr Susnue ue ur alpnord 'sryr1 sazdleue roqlne aql i1 '0t 'qder8ered aql yo lurod ar{l aq }ou plnom leqt lnq'qsa88ns (g) se 'Surrrq atul] preq E aleq lq31ur at{s qrn{M"'JJels ploqasnorl rarl ol pueqsnq raq puodaq ll spualxa 11 'u8rar ulodsap Jaq lnoqe uoqeprads ralunoJ l,usaop ll 'a,roqe pauoquaiu se lnq'alrpe.41e aq lq8p (6)-uor8rtal Jo uoquaru aql-suoseal J€puls ro4'(ado1g'rIN alrasqo ol papadxa $ auo'dre4uoo arp almb) alpnord 'sr14 Sutruasqo u4t{ 'JIN aJaqa sr qder8ered snll se 'sluaul sll set{ Jo aperu sr uoquoru ou lnq'parnpo4ur sr adolg oqe (g) aJror{)'arpnord'sryq 8u;aaururop ari} roJ uoqeruroJsu€rl e 1sa33ns qder8erud srql saop areld ou rr lnq'padse snotSqar aqi Jo asneJaq Suudulal aq iq8ru (y) arroq3 'aurq aql quod Jo lsal aql ,,sassarp rvrol pup uoqedrssr[p],, o1 uanr8 sr aqs q8noqlle 'Jallaq snot8qar;o a18urs slp ol sauoJ ll uar{r'r sraqlo o} salnr aql Surdldde q q aqs plJls moq Surivroqs dq os saop qder8ered aq; dsr.rcoddq s,alpnord 'srl I ale4snlll ol q qder8ered aq1;o lurod aql

J

'6€

we don't know. Eliminate (E). This leaves only (D). Yes, it's fair to say that the mother takes her responsibilities too lightly, and her certainty about the whole affair tells us that she has no doubts about her fihress as a juror.

42. B This question should have been a piece of cake. You did not need to read too much into the phrase "half-heard." Don't let the power of suggestion steer you down false paths. The mother half-hears because she's reading. If the poet wanted to suggest age or poor hearing she would have refurned to those ideas to make them clearer. Here, she wantJto reinforce the impression that the mother has made up her mind so fully that she barely bothers with the details of the trial. 43.D This is an exhemely tricky question. Many students pick answer choices (B) or (C). But the dentist is not said to be a seminarian (a clergyperson). The dentist is uncomfortable, like a priest without the white collar of that profession. The rest of the stanza relates the courtroom to a ship (e.g., the "plank," the "deck"). The dentist isn't compared to a condemned sailor, but is described as though he is one when the mother pushes the fly-speck from the page and says "she will push him off." This statement refers to the way in which she will push him from the plank. It also suggests that she thinks of him as easily dismissed and as insignificant as a fly, and perhaps as repulsive. But the poet does not describe the dentist as a fly-speck. Choice (E) may or may not be true, but it is found neither in the stanza, nor in the poem.0.ly (D)is corect.

M.C

Here you needed to understand that the poem is about the mother and the dentist, not about other people. That is, you needed to stay with the main idea. The phrase in question refers only to the dentist; in fact, choice (C) summarizes it nicely. The dentist is theThackel, wielder of pliers," etc. Yes, some of the items in the list are a bit confusing, but use your imagination. How is the dentisl a'barbe{'? Well, think of the hydraulic chair you have to sit in, or the bib the dentist pulls around your neck; aren't those things reminiscent of being in a barbershop?

45. E Here you needed to stay with the main idea and not get drawn toward a silly answer. Throughout the poem, the mother feels herself to be superior to dentists in general and to this dentist in particular. The mother is a tremendous snob; she considers dentists to be low lifes. Advertising is just one more thing that her-kind-of-people just don't do. You might have had some difficulty if you didn't know the word "propiety." It refers to what is ProPer or polite. POE should have led you to the right answer anryay,however, so long as you saw that other answers all involved reading much too deeply into the passage. 46.D This is a super POE question and it should have been pretty easy. Choices I and II should have been obvious. The mother finds the dentist guilty simply because he is a dentist, and she then persuades the rest of the jury that he is guilty. Using POE you are then left with just choices (D) and (E). So, does the poem imply that the dentist should have been found innocent? Not at all. A11 it implies is that the dentist is an unattractive creep who drilled a patient through the tongue. Does this action make him guilty? \{ho knows? Innocent? Again: Who knows? We're never told with what exactly the dentist has been charged.

175

J

CRACKING THT AP ENGI.ISH TITERATURT EXAM

lll *

SN0llvNVtdXt 0NV SUtrr4SNV:t tStI

I

'8uorm

1r 1aB {qeqold p,nod pue'aruq Buqse,,vr dn pua dlenlce p,nod Buorrvr uoqsanb aql pB l,uplp nod_g ualg 'aflras uoturuoJ pue drouraur,ro prrnq Jaasrre oql lno arnSr; o1 dr1 pealsur pue a8essed aql ol .a3essed oB 1ou o} spl^ a{e+snu sno4spsrp

I I I I I I

il[]VUd

aq;

{Jpq

aq1 o1

T)eq +ua,/v\ nod se 3uo1 os uoqsanb dsea dra,r e spm srql (.swra1,{rnlalrr io dressof rno ur paulJap a.re sldaruor wog 'ruaql uaa^qaq aruaraJJrp aql ilal uer nod n*, n'1nr., pue ioqdupw pue ara8 srural ar{+ augap uec nod arns a{EI :ure8e $ araq lr [ 1nq dpearie salur] Jo aldnor e 1r pauoDuaur aA,aM 'sI .roqdelaur e eapr ue a^eq o1 papJao,,o,(,8uelods leq^. Jo Jo pqr sHl op o1 'aslnoJ;g) ,,'a1o4s1searq dsea u€,, q+rm speal Jaqloru arp aJaqm ,6I aql Suriods alqno4 qJnru oo+ pEq a^er{ },uppoqs nod 1nq ,alualalar auq ou sazrrS uoqsanb aqj .3urpea.r o1 roqdepru e saq)el+e uaod aq+ ararlm pu.rJ pue o8-o1 sem uortsanb qp

lrnq

o-1

da4

aqg A,.6i

'(oo1'araql nod sdlaq q8noql) rDexa dV aql puodaq reJ ssaJrns pue arnseald sSurrq a8enSuel ioy ,nn'.rr.r1 n -dop,raq'(pear nod aqa Btm4dra,ta pue) d4aod_aleprria" xpi ol rapro do1..o.p ol lu'^, nod a8en8uel o1 dlr,rr1rzuas Jo pupr aql +no lurod o1 pa1trn^ n^lng dlpu joplrrorrrrnb ,.rt ra^rsue nod dlaq dlecr;nads srql Suaas saoq deld olrn aruoF-r"1n^

s-

11

*

lprrro.{iar-lsqrF

fuo40ura Stnsea,irul 'punos Stnseanu-sSrmrearu aar.ll il€ aJar{*r tpsrt,, to1asn e punoJ ,seas serl Jazr) 'ruaod aql uI .asrJ arfi uo ,nq *rolr'goilnoraa€ uB parJa ,seas aql ur se aql o1-d1a1er.rdo'rdde aroru uala ro-ap4 Sursu e ur se '.ra1e.rnr Suruadaap ot rcj4 o1 .1.r"*o_ "rroq, Jo qJaA arp osp sr 11 'uorssed ul puu pun:s ur aseaJJur ue sarldurr o1 ,,sosN,, palqs dpoa;rad 1nq'1xa1uor sFIl 14 qral Suqr4s/lensmm .rn r,11 "q1 ,,lrrrriJ.ro*qsal aql"se, aserqd atfi uI ,,saslJ. pJgM aql Jo asn s,Jozry alou'1r pur; nod Jalalaqa. lno Bu4urod rprorvr sr,{'rlsruB asneJaq lsnf pue 'sauqleprurs Buoly .qcadsord luesealdun Jo aJeJ ,q1'.r, q1rros/auo ur alsel ra+lrq e spus auo +eq+ :eapr aFls e;o Sulse.rqdar qsar; e se srroM osle +r lnq 'sroqdelaru Jolres JarlJea aql ol {Jpq sJaJaJ ll,.auriq B,rr}se1, yo srw slas ,"rrjr-d1.rr,, ,\,roq lno Sqluiod qlrom s,ll ra+pmpas;o pr;q1rro*e qllr!\ "8"*, a" p", pue 1ue1d nq11in^ nr^ aq'uaql dgerr.roqdelary dlpn8 punoJ aq III,' a_q lpqJ liarvr BuroS pL rrrr8irryl lnqir",ln-, o1 tm8aq seq lsquap arlJ 'ralemerr r, ,,r*nq. :sdpq rtrelnqero,r ,ure8v .ezuels poor5, roqdepru rye1d-aql-1le^-ol-paJroJ-rolres-pauoop aql samnluoJ "q1yo Bo+rnl,, aserqd

aqg ,,r.4rq 'aturls Jo sauoua^o 3uo'4s qlyvr'p'rynl suearu }I'dlprluassa ,llaM zuEaru snoJaqJal saop ietllvl oS'(v) {ruo qlr^r Ual_ar,am,ilocl Bursn,snql .(g) aleurunrd';;h;, ior,,.rorlnlh,;rl ro, druearp Jo luaruoru e ut an8uo1s,1.tat1nd qq palllrp .1d aqJ s,araql lsrluap lpulg'.(q) aleu -lum'JEruuru € olrT aroru spunos oH rrpr{J aql plmorE }ue}srsse slti pasBqJ aH .oN Zplu4i lsquap ag1 s1 'aldnurrd peq-p = pPq-JIer{ aql 3u1snl1du1s dq a}ernurTa plnor no1 .af,ror{r aql al€u.ru4la ol lrODI o+ paau l,uop nod lnq,lear8,mouT nod;1 isnonwa s,leq,M.qero^ aroru purJ nod (q) u1 .3ur1e1nr1er +ou sem lsquap aql,(f,) aleunurTa uosear,*n, roC '(g) alpunurTa os '8uorm s,leql .an8uol s,luaned qq g8norqi panlrp "ip-lprn*, dlaler"qpp o+ p'r{ plnoll }sl}uap aq} 'pailor aq ol (g) rog .ssan8 e rol ",rnq (y) aireal p,nod +ueaur iror"qr"1 'an8uo1 l€r{1w Mour },uplp nod;r'o5 s,1ua4ed e qSno,rql lq31.r qIIJp aq }€r{l a^ol q}vvr +r Jo +no os Suroq srq +noqe oo+ duurry dlTrs Surylaruos s,araq+ pue ,sa,r.1aqs aql ruoq urr8 qpai ralseld paplotu alqlvr rleqJ arp ptmore 1*lsirre stu Sursnqr dlerruor s€ uaas aq plnor +srl -uap ayl leql_des o+ (V)ul luearu snuatpal lerlm MornI o+ a^eq no1.peqJp ;o a1d'tmd

aq+ pue

l,upp

irocl

o+

JIaEr spuar 1nq1,rorlr"nb n yo

a1d-e"" pooh

i

orln

iI

.g?

= peq_JIEr{ .aldrue

,,1i

-xa poo8 e sr uoqsanb snD pue lsar aql olur drepqero^ aruos Buoleaus yo xl^ n'inq

sJg v

.r7

50'

51'

52'

D

c

D

This is primarily a t-erm question, but you could have arrived at a correct guess without knowing that "poetic jushce" refers topunishment that reflects the crime.lFor example, a counterfeiter,buying an expensive old painting with t;g"r money only to discover later that the painting is a forgery') In the seventir stanzZ,tnu po"? a"rcribes the dentist now in the position of a patient, gnpPing the arms of his chair and being most uncomfortably drilled. This sort of reversal atso rils under.the category p""ti. justice. If you were unfamiliar with the term poetic justice you could haue i-rtiu"J "r iiu p.rr*uy good guess by reading the question carefully. It refers to "Mother'r tr.ut*L"i;if the dentist. In which stanza is ttie mother most directly involved with the dTlif, himseu (and not simply the legal process)? In the seventh stanza, where she "strapped him *r, to arru him awuli;'rrri, should have made (D), the most attractiuu g.r.rr. "ria.rrtanding Guess.By the way, there,s another more technical definition of poetic justice, which"grs witt prorauty not use. we cover that definition in our glossary.

There's our old friend irony again. In the phrase "Mce Mrs. Nemesis,, the irony is not very delicate; in fact, it has almost iecome irony's nasty little brother, sarcasm. To answer this question, it helped a great deal to know that ,,nimesis,, a is an archenemy. 0n the poem, Kizer actually refers to the Greek goddess, N.-erlr, *no represented righteous anger.) If you knew that a nemesis is an a"rchen.*y, o, negative (which you could have figured out from context), you could"uu.,lusisomething rtuuJ..uro"ed tha-t,,NIce Mrs. ,something nasty"' contains the kind of contradiction that makes for irony. Barring that understanding' you should have worked with the terms yot, k r"w and,rr.d pOE. All the terms in the answer choices are covered in our glossary of t.r_r. These I,II, u questions are made for PoE. A{ter reading through the items, you should have gone back to the final stanza.and reread it. Then to'ot ut the items again. \{hich choice is easiest to decide upon? Choice II should look weird-eir-inut it. The stanzadiscusses the mother's idea of God; voltaire is an afterthought, anJ ail that,s said is that she finds him "indispensable'" I{hat voltaire's views.le, the pJem doesn't say (and ETS does not expect

you to know voltaire s philosophy).wilh item Ii go.,.,.noi..s @)'and G; u.e go.,e as well. what about item III? social decorum refers to pori"t. uurt"vror. In the last stanza, the mother mentions that God instructs in 'hygi9n3 and deportm.r,ii' thut is, in necessary social graces. Item III is a keeper. Even if you did"n't know what decoru- meant, which makes more sense: the mother believes God agrees o. disugre"s ? rf yo:,ug"t A+;;rar drift of lith h91 the poem you should know that ihe Mother thinks God"shu.es her views. Keeping item III means you can eliminate (A). All that,s left are choices (C) and (D). okay, teys.toot at item I' Here you needed to read closely. she says coJir- i"aispensablei; r6uit gooa enough to justify the.'deeply held" part of item I. \{hat about .,unsophisticated.,, Is it a sophisticated conception or fut. oi"i"e to think that God cares about hygiene? Not really. Furthermore, Kizer's comparison of the mother to ,,true iJoh,urr,, reinforces the unsophisticated idea. Does the mother think she's ,trrroprur,i.uted? Not at all! She thinks she's hot stuff reading voltaire and all! But the q""riio.iao"sn't ask what the mother thinks of herself' It asks about what-the poem says about her. In the final stanzaKizer has some fun at the mother's expense. Iternl is a keepel, which makes the right answer (D).

I78

I

CRACKING THT AP ENG[ISH TITERATURE EXAM

6ll a

SN0tMVtdXt 0ilV

SUi/rtSt{V

:l lStI t)tIlVUd

'no[' palzznd (g) arroqr ur ,,a)uare^rqlup,, ruJal aql Jr ua^a Ja^flrp +q31, atR nod uapo8 a^Eq plnorls gOcI os ,a1equ41a ol dsea dlpld uaaq a^eq p1not{s pue ,paqBsnf aq ol arua4xa oo+ are lEq+ salqs puoqorua dn 1gr IIp saJrorp +JarroJlr aqJ .lla,ra ,rr11"r*nq1'3* -pnpuoJ dlpnpe aJe uaql ,*unlrrnsrrT_qrs Jo Jarprau 1nq1d"oa rq1 pr'rn aJrI ,y-ol ,,1rp.rrp aql 'pnpuoc radord Jo asuas dloous s,Jarnoru aql qselJ r.11 ,o*n"q Bull.raprm'rq1 ,", Jo

*

ol srayard razry lnq'8w>lroqs uaaq a^€{lqep pasnrue,(f,) ot "bunn+t.r ".11'arualelrqrue dn sppe Srmrosear srll Jo 11y'saruinbazuor rr8erl nrtnq 1,.r#op apnrp+e JarrEAe) s,Jaq+olu aql os 's1a8 aq ler{^' sa^rasap oq^ daa.rr e a>lrT prmos saop lsFuap aql lpr{l1rny ,q1 dq paua -uos $ Srmn aloqrvr arll 'l€JrlrJJ dnq8l aJe sluaruarers asaql .r+lap aqr Jo uorsr^ s,Jaq]oru aql salelal aqs uarl^ rou pue ,,'srsaruaS _sJlry a)IN,, se Jaqlou aql ol sJaJaJ aqs uaq^ lou 'os dp.r4ua lou lnq'raqloru aqr ol Jpaqi"a-rr r,'/;y ,.rrf.rp"p*turre; [s,rarporu aq1] 1o {JaJr\,, aln an)sar or a4e+ro^ uodn qlec aqs se uala (arueS0rre pue) ql*rar r,rrq1o* qlrlvr passardurr s,aq5 'uoqentls aql qlllv' unJ serl .razp1 "qt 'dpso14i 'i""*si"lsassed ,Jnr.,+r,q lr s^oDI razry dlqe*uoqe pa^eqaq seq Jaqpur aql spJ'pue+s IeJrqla Fuorlua^uor dg .an4 s,ll puv 'lsnuap e 3u1aq roJ ueru e SrmnuapuoJ rr rreItm pnn qrr".1d11ard Suraq r, ,;qro* aW Sqlqql gas.rnod punot dlqeqord nol ,*aoj nqipnrr noi ,y .uo4sanb auot p q sHI 'pauoquaw Jalau aJe slarJaq snor8rar s,lsuuap aql iramsue paJJoJ aql $ qlrM uar are nod 1u'pua aql rrn rr rryl raqtuaura.r) alernrurTa t'(iuopsanb raarxg ol sar4l asoql Sursn saJroqJ aq1 q3no.rq1 {JoM ol sE,/vr op ol peq nod 11y derue uorlsanb srql a^e8,,iJaJlreJ qlra paqlnour-ua11o^l luaued srq lvrou p,ry 7...rrrrol,qlqeq puos.rad 'arue.readde ,aJrlJeJcI /,uorssayord ,,n4q dq pauoop ,paruoop ,paruoop aql

,;^;il;r:"fi

iI

'99

V 'v9

'alnJ leL{l ol suoBdarxa ol sDIII srfl 1nq,an4 s,leql 4lpo* daql (s;p.ro,u arp ol lxau pareld aq plnor{s srarJrporu l'rn lq8nel.tnaq oqn xlqnfl-d ,",r,o1 .1,.rfp xrgl uaq/r lr sa{ll sJg .raqloue auo o} asolJ aruoJ o} a^eq lou op qJa^ puE pafqn5 .nod ,vrorql -"-lT *r^T-1] 1dans,, pus,raqlol/{,, uaom+aq aualralrn sprol,4,leralas terll q uoqsalb 1lllg l'+"o arB letD sarualuas lnoqe suoqsanb 1se

,q*o,,,ain1'drp;i;;;'{;#d;6iffiffi;;?#,":Jffi:i,il'#;,:# aql uaqa 'auo sFIl a)rl suo4sanb reuruerS0pnasd ro ,reunuerg {se ol

sqr srg d11rya.rer

aJualuas aql ppar pue ruaod aw ol {Juq auoS aleq ppoqs no^ raq}ohtr sr ra^sue aqJ e,,Jno 1darvrs,,^oqar $lse lI ,flJerrseg .uoqlanb ,ffn pir*Srrp n

lrrf ln .frnr*l reunuer8 e l,usl dgear 1r'IJEJ uI 'uousanb"op""qrra*.reuure.rS dsua dpa e uaaq a^eq pForls srqr

g

.gg

HOW TO SCORE PRACTICE TEST I Section l: Multiple Choice number

-(1/+x-) =correct number wrong Multiple-Choice

Section (See

ll:

Free Response

if you can find

essay

(out of

1 9)

Score

a teacher or classmate to score

essay

(out of

2 9)

essay

3

your essays using the guidelines in Chapter 7.)

Free-Response Score

(out of 9)

Composile Score Multiple-Choice

x 1.23 = Score Weighted Section I Score

x Free-Response Score Weighted Section

3.06

=

Weighted Section

I Score Weighted

AP Grode Conversion

Ai cTd" 107-150

5

93-106

+

73-92

J

43-72

2

042

1

t

a

,,

Section

I

II

Score

Score

Composite Score

U

tsol olllroId

tSlm z rstr illl)vud

snorqn^

aq'

ua^earr

'aprm "d:Xt^'fi"if;:n " Jllv+waad','-d;;'ffi:l:]'.T'jfi:':i'4ru"; +noqe punor rvrou sJa^rraasorl'

l! q"gr*; "ririr,.rnqrnq uopuor q araq lros re8jn,r aq,"in+.r'oy "q1 uro.r; a8en8uel rno pasueap 1gp; "pn*aaeq.daql.,fro,i"roy prn

"'Y"i:ff[,1":H ;::r: "q, o+ x'",,J"* iJ1n,*"q, .rno ;o slaod

aqseN seurordl_

;i; il;;;tJ#^T,H;'ffi ,,?#:;";

*'

-::!i"1,""'.i,"J:;il::Hffi;$TuJj:#:"iii , , '#ifftH'ij#:ix#:f#,:.,",r:y#"J:,#f:if Iailaq sr rpnui oD s -ry^11+.r, r" ,",r'u .i"r araq+ toalaq^ se ue Lrlaod ;1n.ii;;;il*rffffi"i? ar'l srq .' ?:-.T:f +r

nasurrq qro,oi

.

ueql €snec s'auo

lletd

,"q,,ufr:;il;*";il; ,"d^n1t*3+

o+

Jir'"# ft".il.rrqr;;"rr',J;,di:i$I;J:fr,1, ra-{n lsn.' o^ ,,^J.r'au.rr"raq qr'^

^aN,,

raded

,r.,

atrsea,r

'p"o* " ,.,",, "o "q, j:J: j#+",Hr,Ttr#Jf#:,:::',{"fl#,,,,"T,lrffi :*il:ii*#::{*}:;'"'i#

r:JifrT f3,11$l:,15 "*;;il:H"1

uj 9^eq o+ x'"^

=r"s"J vJ "1"!a

,,q

rllr*fi'd;r:rrffi"#:;fy' .' ;#;:ll:if[#3j+ii?:"#*g#'''

.".**lll,ti#i:"ft

'""ff y,sn'*;i;**fi '":,,il3

:;l{3#*;i1ffi'"'"'*rh,l'lffi

o,, ;t.x1;';"fl#:'"i'I,o.,,,,i',*il;;:Jffir."$ #S:i'.{""THi3,"'ilf:i3i""Ii'i:l#ffi 'Tot t rno arou sril arots t.?"ro., ,rnu,t,lJ,"fit ;:|fi:i:;Hl,",yll:it4 "q ;;i;;s#t"o "o au -ro_parrer Lpns sdaax _ ul xru"a,j,"q,,,"-r"d$###:,T.f1ij#t#jjf '"' -',1h5tr;#,'{;;F;#::,i:t:ifrffi ;: e

j:-t1" sreJlrd tarur aqr ro3 'a;o;41 ;;*;;; o',:::0""; oJJ"y:::q.::ro.{*N'il#::j*:i ,

ipasno.lretql"i,t_::3.r:""*r".1"s"#"rsq,

";:j':;fj:"'x5'i*{1*}il::+';'x'ffi':H;'u ,.;;,1l" *n1n3'n nJilj:iT":',fffi1'":ilffX wlil) os)

,

'"

plro^'{"'nd;#:.ti-i".r idifiH:il:ill,' "',",T*'Jfi,1:g::"t?:i.{:"0"ot*r;,rilll.**r ol q,eor;[il;;;Y ,,*f;lli:::ff#:df"f,#3"|.iit*?Y"u,r, ,r,oo]'in,u.nru'r 'a^p ur t.q l', 'll,!ot'n'u'nlu'r \p au!., 'ou

,#;1f ;Iffitffi'rf,?1:_ _ o:?jo,'";il"::"1ffi:,'l1fjjil,f.:"*,:f i*i:t

nn su!pnn"*"Lp,,o^ q1"pn 0"" ;,,,;::1: aloq^A e g+asrdsap,suad_4aw;q;il;;:;paep srq a^eq o1 adoq :qiJ"l,

sd;liil:F:#Xiifr"#llXT , t"o;:;rfrITJ::"Ti"lITj"*"'n rvt "'JT"p;i',oq

q3ry .{ureyur "H:'i":'#,l'J:"T:jiIl'lH,",T:*r*r[r*:"::,: ",t^:,-P.f"

J.ia

,"tte

o,a.,,

"'aqr

e,',-;;,. ]3T-n_?

"^""qi"

l NlaodJo sa*uaua aqr qll 4 .__^ - -^ saluraug dr1ao4;o ^^*yu lsure8y a.ruJa^'J uv

,,,, n,_,".T,".T:j"xTI,,,:H,"##"r^:r.:&T{ffi

.prro

rauv

Burpuods;

ard+so*L"'''"'llll;J1fi1"Jil:fiil'Jl::l"H"r::jj#'JH:ff#":,il:i;fff*Ti,,'"""Xx;1ffi rnoq l_arurJ I NONf,iIS

NOf,tISOdhIOf, CINV AUNJWSII'I HSI'IDNII

In lines 34-39 London is described

In the fust paragraph, preachers are accused of all the following EXCEPT

1.

as

(A) flooded (B) a damp, rainy city (C) the main influence on the English language (D) a cultural garden (E) an important port city

(A) plagiarism (B) stupidity (C) dullness (D) eloquence (E) laziness

The main idea of lines 39-48 is which of the

"Satumist" (line 9) means

following?

(A) astrologer (B) nymphomaniac (C) depressed and depressing Person (D) pagan (E) foolishly optimistic person

(A)

People are motivated by concern for their

reputations. Poetry is fair to the virtuous and the evil alike. (c) Poetry is inspirational. (D) Poetry is most atlractive to atheists. (B)

(E)

Poets are very judgmental.

Lines20-27 argue that

(A) (B) (C) (D) (E)

poets must take second jobs to make a living most people don't respect poets there are too many poets poets have to work hard to present consistently fresh material poetry books are never bestsellers

"New herrings, new!" (line 24)

(A)

refers to an implied comparison between the writers of new poems and the sellers of fresh

(B)

suggests that poetry is slippery and hard to catch the meaning of, like fish

fish

(C) implies that poetry is just another commodity (D) implies that poetry grows stale rapidly, like fish (E) compares poetry to rotten fish

I84

:

(RACKING THE AP E}'IGTISH I.IITRATURE EXAftI

7.

Who is Salustius (line 49)?

(A) (B) (C) (D) (E) 8.

A French poet Sidney's nom de plume The Roman god of poetry The King of England The Wife of Bath

What is Bath?

(A) A state of sin (B) Acharacter in Chaucer (C) Amarriedman (D) Apoet (E) Atown and spa in England 9. In the last paragraph, poets are said to be

(A) (B) (C) (D) (E)

lawyers mayors chronographers townsmen angels

like

98tl z lsil I)tllvud

uo+IUt (tr) o+eLI (C)

snqsnlps ()) dauprg (g) aqse11 (y) o1$gssarau,slaod yo anssr aql

o"rr"r

r."1fTff

azr.re€e1d

r__suoruJas ltFneaq pue a+e4s.nlll snornellrl nrfl ,rnrr"q*"

aln &ina snory4l aq1 a8ernorua

.-^:?:d*r 'uraao11ol

"*

r"

slsturyes

:

!.o

sralrralearD Z(9

aqD ,,sauI Ip,, arp lpr{r\A .ZI

o, /r\

r; iri oJ (V)

salLuaua

u"*ft?x1#r"j'fiT3;fpa; r,

rr€

(g)

sraqceer4 (y)

o1 (g) oJ (a)

gi

lnq aruarpn€

srallrm pea6l (J)

.n,

suoulras (S) acuerr,alar 1qj

^dpo *-io,a

(S)

slooC (C)

"*"J;X (v) uoquadar

;;il i;Jn"r,ff ;HiJff:.##""jff ,,

s,,,(4aoa (A) stueo4 (q) spEIIPg (f,)

rorpne

aql

(A)

,Z

q .II

qao4 (y) i,esofi,,

Jo +uaJaJar aL{+ sr

+eqr

aull

aqdo.rlsode (g)

uoperalrye (q) . xopered (3) eradaoleruouo (g) roqdepur (y) ;o aldruexa ue q 0I aulf .0I

Ouestions 16-28. Choose answers to questions 16-28 based on a careful reading of the following poem by john Donne. Let me pour forth My tears before thy face whilst I stay here, For thy face coins them, and thy stamp they bear, Line And by this mintage they are something worth, (5) For thus they be Pregnant of thee; Fruits of much grief they are, emblems of more\Atrhen a tear falls, that Thou falls which it bore, So thou and I are nothing therL when on a diverse shore.

(rc)

(A) (B) (C) (D) (E) 17.

So doth each tear Which thee doth wear, rs) A globe, yea world, by that impression grow, Till thy tears mixed withmine do overflow Thisworld; by waters sent from thee, rny heaven disolv'd so. :

O more than

moory

(20) Draw not up seas to drown me in thy sphere; Weep me not dead in thine arms, but forbear To teach the sea what it may do too soon. Let not the wind Example find (25) To do me more harm than it purposeth; Since thou and I sigh one another's breath, Whoe'er sighs most is cruelest, and hastes the othels death.

1g.

the end of a romantic relationship death the separation of lovers the end of the world a pleasure cruise

Lines 10-16 are an example of

(A) (B) (C) (D) (E)

On a roundball

Aworkman that hath copies by can lay An Europe, Africa, and an Asia, And quickly make that which was nothing, all;

(

16. The situation described in this poem is

paradox dramatic irony metaphor metaphysicalconceit dramatic monologue

Line L9 is an address to the

(A) moon (B) world (C) poet's soul (D) workmen (E) beloved 19. To what do lines 14 and 15 refer?

I. II. m. rV.

V.

The speaker's tears which reflect the beloved The beloved's tears The beloved's clothing, which has been torn as a symbol of her grief The ocean, which is salty like tears The rain on their faces

(A) I (B) I and II (C) I, II, and III (D) I and IV (E) All of the above 20. Which of the stanzas do NOT include images of roundness?

(A) (B) (C) (D) (E)

I86

T

(RAC(ING THE AP ENGTISH TITERATURE EXAM

Stanza

1

Stanza2 Stanza 3 Stanzas 1 and 3

None: All of the stanzas contain images of roundness.

rtll

u

lsil l)[]vud

prmor8 aql (s) areld luaragrp y (a) adorng ()) IIAH (s) uaA€aH

dSololeqrsa

(S)

eu€alu (0 aqD ,,aroqs asralrp,, saop

uqlallered (q) .(uor1 (3) lus€Jres (g) aloqraddq (y)

sJaAoI Jarpo >Iaas ol l.{8F aqJ lau8 .raq ure.4sa.r o1 razvrod aq1

,,uo41sodsrp drrtms,, ;o alrsoddo aq1 ddder4m yo arrqsaSSns sp uooru ar{J

G) (c)

ol

razr,r.od

aq1

aler.rdordde ue

ION

(v)

8q1aay alq€nle^ e sr;arr8

$qM 'lZ

dw

eJeJaJ //qJr-qA// prorvl

are daql asnecaq Srr.nparuos rl+Jorvr Srmlsa.r;ar dleuopoura are- s.rea1 dtu ()) aJ€J Jno^ lf,alJar daq+ asneraq Stln4auros Vlro,r,r are s.real dru (a) srual dru Jo lles arn rllrom 1ou a.re nod (v)

uearo

'gz

(g)

2uraod aql lnoq8norql paq€r

$

qrlqM

'EZ

aql ())

aql saop l€q1vl

ol'tlaLqIwI

'ZZ

sJaAoI

uooyq (q) sulof, (f,) saqolD (g) srca1 (y) -sns

plro,|vl

(A)

rnod ro; are sreal dru (c)

t a11

}o

il€qPrmoraw (a) sardoS (v)

a>1es

se paserqdered aq lsaq ueJ

sr 3uuvro11o; ar{l

quau4uoJ at{I G) ueuDlJo/r at{J (c)

z9z-02 saul tn pa^olaq srq ol aqrJrs€ raleads arp saop

G)

uooru aq+ ur u€ur aql Jo alelTlot aqJ (c) uooru ar{l;o adeqs prmor aql b) saprl s/u€aJo aql pu€ uooru ar{l uaauurpq dlqsuouelar aqa (s) ssappoS e;o ramod aq; (v)

20z-6l sarnl roJ uollurJoss€

aql (r) urr,q ilH o1 razvrod aql (s)

s+ualuala lern+€u aql aJuanlJur o1 rartod +Jeaq srq >leaJq

lBqM '72

's8ur1aa1

Jo asn aql q8norqt pa^olaq aql sral+Bg ra.4eads aq1 'rueod sr-rp Jo srogdepru papualxa ar{l uI '82

(v)

d.raSeun yo saddl SurmolJo; aq+ to qcTrl\A 'gZ

arn uaart+aq puoq 1enxas aql 8uusa88ns Ja+Br\ .sJa^ol ar{l

(a)

Jo ssaussalq+py Sqg1qs aq1 3u4sa33ns ralerr 'pua ou aleq salJJrJ asnpJaq'sralol ar{l uonm luaueurrad pu€ f,nusoJ arll aroJarar{+ Jo pue'a1c4e pa;red e 8upsa83ns ssarrprmor

(c)

(r)

'uorle,redas ,sJaaol

aql Jo saru€lslp ls€^ aq1 8unsa88ns aqo13 aq+ (s) 'uoE€nlls /sraaol aql aq+ 8ucsa88ns ssaussapuom (v) ssaussaladoq ;o

;o sa8eun parn€lsns s€ paqrJJsap aq dlale.rncre lsour ueJ ruaod snn u1 draSeun aq1

'rz

Ouestions 29-39. Choose answers to questions 29-39 based on a careful reading of the passage below. The passage, an excerpt from a short story by Mary E. Wilkins Freeman, describes a woman about to be married after a

long engagement.

(50)

Among her forebodings of disturbance, not the with regard to Caesar. Caesar was a veritable hermit of a dog. For the greater part of his life he had dwelt in his secluded hut, shut out from the society of his kind and all innocent canine joys. Never had Caesar (55) since his early youth watched at a woodchuck's hole; never had he known the delights of a stray bone at a neighbor's kitchen door. And it was all on account of a sin committed when hardly out of his puppyhood. No one knew the possible depth of remorse of which (60 ) tkrs mild-visaged, altogether innocent-looking old dog might be capable; but whether or not he had least was

Every morning, rising and going about among her neat maidenly possessions, she felt as one looking her last upon the faces of dear friends. It was true Line that in a measure she could take them with her, but, (5) robbed of their old environments, they would aPpear in such new guises that they would almost cease to be encountered remorse, he had encountered a full themselves. Then there were some peculiar features measure of righteous retribution. Old Caesar seldom of her happy solitary life which she would probably lifted up his voice in a growl or a bark; he was fat be obliged to relinquish altogether. Sterner tasks than (65) and sleepy; there were yellow rings which looked (10) these graceful but half-needless ones would probably like spectacles around his dim old eyes; but there devolve upon her. There would be a large house to care was a neighbor who bore on his hand the imprint of for; there would be company to entertain; there would several of Caesar's sharp white youthful teeth, and be Joe's rigours and feeble old mother to wait upon; traditions for that he had lived at the end of a chain, all alone in thrifty village be to all and it would contrary (70) alittle hut, for fourteen years. The neighbor, who was (15) for her to keep more than one servant. Louisa had a choleric and smarting with the pain of his wound, had little still, and she used to occupy herself pleasantly in demanded either Caesar's death or complete ostracism. summer weather with distilling the sweet and aromatic So Louisa's brother, to whom the dog had belonged, essences from roses and peppermint and spearmint. had built him his little kennel and tied him up. It was By-and-by her still must be laid away. Her store of (75) and there would now fourteen years since, in a flood of youthful spirits, (20) essences was already considerable, pleasure of it. mere he had inflicted that memorable bite and with the her distil for the no time for to be exception of short excursions, always at the end of the Then Joe's mo*rer would think it foolishness; she had chain, under the strict guardianship of his master or already hinted her opinion in the matter. Louisa dearly Louisa, the old dog had remained a close prisoner. It loved to sew a linen seam, not always for use, but for (80) is doubtful if, with his limited ambition, he took much (25) the simple, mild pleasure which she took in it. She pride in the fact, but it is certain that he was possessed would have been loath to con{ess how more than once of considerable cheap fame. He was regarded by all the she had ripped a seam for the mere delight of sewing children in the village and by many adults as a very it together again. Sitting at her wihdow during long monster of ferocity. Mothers charged their chjldren sweet afternoons, drawing her needle gently through was But there (85) with solemn emphasis not to go too near him, and peace itself. fabric, she was the dainty 60) the children listened and believed greedily, with a small chance of such foolish comfort in the future. joe's fascinated appetite for terror, and ran by Louisa's house mother, domineering, shrewd old matron that she was stealthily, with many sidelong and backward glances at even in her old age, and very likely even ]oe himself, the terrible dog. If perchance he sounded a hoarse bark, with his honest masculine rudeness, would laugh and (90) therc was a panic. Wayfarers chancing into Louisa's (35) frown down all these pretty but senseless old maiden yard eyed him with respect, and inquired if the chain ways. were stout. Caesar at large might have seemed a very Louisa had almost the enthusiasm of an artist over ordinary dog, and excited no comment whatever; the mere order and cleanliness of her solitary home. She had throbs of genuine triumph at the sight of the (40) window-panes which she had polished until they shone like jewels. She gloated gently over her orderly bureau-drawers, with their exquisitely folded contents redolent with lavender and sweet clover and purity. Could she be sure of the endurance of even this? She @5) had visions, so startling that she half repudiated them as indelicate, of course masculine belongings strewn about in endless litter; of dust and disorder arising necessarily from a coarse masculine presence in the midst of all this delicate harmony.

I88

ff

CRACKING THE AP TNGLISH LITTRATURE TXAM

68tl Z lslr llll)vud

'u1t4'ol

daql

(A)

'saulap +sa^req I€Jnr luarrue Io slen+rr ssalalurl aql azrloqurds daql

(y)

palualuoJ s,€smo.1 ale4suoruap '7€

Z(Z Pue 1 sqder8e'red) sarrrlrne plor{asnoq s,e$no-l saqlJrsap lsaqluarua}e}s qcTt{M '0s

uorsnTle (g) uo.rorudxo (6)

aPuns (3) roqdelaru (g) z(uorr (Y) yo aldurexa uP sI €7 auII ul,f,71md,, prom

saluor{ drquac-qluaalauru uI pooJ lad 1eorci,fi

aqI

sI

(a)

q

(C)

slruraq ur dreqrlac a8ernocua ol luearu aarleurd snor8rla'r e;o ped

u€urom 1arur'qsulas e sY (a) u€uroM luaJouul'Parallaqs e sV (C)

luaunlsnrnd su;o Ped sr (A) dlra,rod s,eslnoT sPagar (Y)

JesaeJ'zE

a^oq€ aql Jo auoN G) a^oq€ ar0Jo IIV (o) AIuo A b) dluo 111 PUP'II'I (a) 1

(v)

p1o u€ qJ€al 1,uer nod

n

dpo 11pue

$lJrrl lrlau 8op

,,'lno trrrt{ a>lel ol Buro3w; $91)

pue Sur8ueqrtm dq padde4 aq ryD aldoad 'AI

dep aruog'araql dn pap urr-q daal o1 1am lqBrruzvrop s,lr pue,, 'des pporvr ar{ ,,'uMol uI 3op PaJn+eu-Ja$aq € l,r4B alat{J,, 'sl€^ra}rrl 1e r{1qmo1 almb ralleru aql r4 uorwdo srq Surttmouue 1da1 1nq'palsrsap aq leql

roradura Superrul

Srndrretu 'm AIaIDOS

fl

sleurrue o1 fi1anrc a8ueqc auroJlam o+ aJrl paqlJJsurnf,4J raq 01 pawolsncJ€ oo1 trrvr.o.rS s€r{ €slnoT

Iarur € Jo Jol€Jrpur ue

'11

paruPF '

sgl l4 pasn sI r€sae)

1o d.ro1s

uml ps

o1

paldruaBe (001)

apo4s aH'sP,lvt aq se uln{ /v\es'ssaupMalqs pue asuas

aq1

qrlt{11 Z:I1UJ ar€ sluarua+e1s Surrvrolol aql Jo

os zvrarS €srno'I'asool

uaAa pu€ €tmrrer"r;o rolu€IJ Uos s,€smol;o alrds uI'peaq aql uo ury1 pa11ed pue urlq o1 dn dl1ue1p'r

'I

+eql €epl aq+ acrot -urar o+ a8essed

'62

pue aae*reur erm'eorddB 'raq yo lq8noqll"#X; aldurrs qq Su.nlcrmru 8op p1o aqt le palool es1no'I 'sauoq puE qsau Jo lalp ,{reurn3ues pue Surleaq qlrzvr radural snora8uep srq parrl Jelau pu€'sa>1ec pue qsntu -uJoJ Jo aJPI Jparse uo rur-q pal aqs 'runl reau oo4 oB ( g 1 1 ) o1 lou aldoad paur€M sdervrp aqg dlporal s1t{ uI qlp} leer8 peq aqs IIFs lraq qilrvr a11ua3 rfiarr sdezvrle servr aq pus 'raqloJq peap Jaq o1 pa8uolaq per{ eq asn€Jaq €bp p1o atlr to puoJ d.raa;lasraq s€ra aqs'qled qq ut Surpaalq uarpITtIJ lueDouuT lvres ar{S 'a8elpn papren8un (p11) pue ppb aq1 q8norql a8eduBr aql uo resa€) Jlasraq ol parnprd ar{S 'auo uI pasry dlalaldruor arotu aq pFoqs suorssassod pue slsaralur Jrar{l uaqrvr's^€p asaql Jo auo'lou plnom aq leql adoq a1++11 d'rarr peq €srno-I

sEapl parnurcxaun

ue dq paranbuoc Suraq a1q sr

uI

"t"

spagar (3)

(7 qder8e.red) lalp,,cpacse,, s

sV ())

ueurom qsIPIIqJ'arueu e sY (g) ueurom SurraatmuoP'ragrq e sV (V) zpazlralJer€qJ esrno'I sI .{!(or{'srural IIEJa^o

a8ueqa u€J pu€ la1p dq pa+JaJJe ar€ sJounq e

ueurol!^' qsqooJ'paualq8rr; e

'€€

uoqrsodsrP

,{ppoqleqr Lroaql,(m4uar-qluaapqu

G)

Jrlsrup passardar s,esrno'I aleJlsuouap dn 11 (O) 'ssaullr l€lualu s,€smol ale4suoluap daqr (c) 'araqds l€rn1lnJ aurunual dlpuoPlPe4 e ur uo4drosqe

sI (7 puP €

aql

daql

'snrua8

Sutreuaur (A) StnPoqa'ro1 (6) arusuad (3) lueu8rPur (S) I€crrlt€s dPua8 (Y) sqder8ered) resaeJ;o uondrnsap ar{r Jo auol

s,eslno'I

aql eqr.rcsap Jo sanlel I€uoqrp€r+ lsaqSrq

.I€

paJorunq-poo8 qq qlrrvr'.larrarvroq'ao-['snoturoua pue dppep pDIooI pue saqllno radord umo sr-q $ol (s6) "tt8nrt aq lBql os 'umr pamop€qsJa,ro uorlelnda'r sru'paurcqc

35. The contextual meaning of "sanguinary" fline 117) is

(A) expensive (B) feminine (C) masculine (D) vegetarian (E) bloody

38. Which of the

(A) It shows us joe's down-to-earth,

best describes Louisa's beliefs about gender relations?

(A) (B) (C) (D) (E)

Men and women naturally belong together. Men and women should remain separate. Men bring chaos and possibly danger to women's lives. Women help to civilize men's natural wildness. Men are more intelligent than women.

kindhearted

character.

(B) It syrnbolically

shows us Louisa's fears of the

future.

(C) It serves 36. Judging from this passage/ which of the following

following are accomplished by the

Caesar vignette?

as a slrmbol of what happens to those

who refuse change.

(D) It provides a humorous

satire of small-town

concerTrs.

(E) All

of the above

39. In line 46, how is the word "indelicate" used?

(A)

To indicate the differences between Louisa

(B)

and Joe To indicate that Louisa considered her thoughts

(C) (D)

To indicate the coarseness of foe's personality To indicate the inferior quality of Joe's

(E)

To foreshadow the vision of Caesar's rampage

inappropriately sexual The contextual meaning of "mild-visaged" (line 60) is

cl.

(A) having a calm temper (B) having a gentle face (C) having an old face (D) being confused (E) having a kind mask

I9(l

:

CRACKING THE AP ENGTISH TITTRATURE TXAM

belongings

--

l5l: u tsll t)tl)vud

u8rsap adecspuel ru€arp Y suer.rela8a,r yo sualqord ag1 qsad uapre8 Jo uoqermurla aqJ

G) (cr)

(f,) (a)

II reM plroM (v)

Zaqrnsap dleralq ruaod sr-ql saop

teqM

.0?

deru rze1rtr ppb aq1 prmo.rSlaprm passeS (pg) uaesun arp ol paluasuof, ile p,{aql {I*o Jt 'daa1s dru ur Iarreq aql Suoplq8gs 1 ruearp I'urroJ dn-padumq slq ltmr{ I lq8Tu IIV dep rage,,(ep.ra1;e dep dpear pue pe>IJor aru

sdaal aq'zrto11a;

Jo aldur€xa poo8 e sr (9I

.roqdepru ursrrul uorourdxo uo4rsodelxnI

(c) (J)

xope.red

(v)

aql) ,,saqard treln

s1sa33ns ruaod

(s)

'saso.r SurreaqJa^a aql rrr umop palp aH 'aJeJ s,TJnqDpoon +sal${ aq+ uo peaq € MaJp 1vtou

'JelJllrts All€puassa sr ef,ualorl IIe leql qsaSSns ruaod aql 'sJaIIaq snolSqar se IeuoBBrJr pue 1euo4orua se are sJarlaq pc4qod 1ertr1 s1sa33ns ruaod aq;

(q)

())

lercos lrerodrualuoJ pue dqdosol.nld rzeN uaam+aq qalpred smerp uraod aqg (g)

'uoll€unuralxa qroldxa unural Jo Jrrolaqr rzeN arp ruaod aql (y)

Ip4uassa

q pooJ

oXXt"fii*

G)

uzrto rnor{ Surqe.r

(f,)

aJrI

^ep^Ja^a

Jo sal$€q aq1 lsure8e luep8rrr-.rorra aq lsnru azvr plJorvr arp JaAo 3r+{n+ are sl€rurue aril se paqrrJsap aq lsaq ppor,r, ruaod alp Jo

'slorreJ aql Surpeaqaq's+ooqs rtoJroJq aqt Surddru qoled alqep8aa aq+ ra^o {ool uaq+ pu€ asrnoJ Jo Jailsur e se splo8uetu agg uzvrop lqSnoqt{e\a @1) 'q4erJs o1 dn sn;o il€ 'r{clors aro+s-a}els pue salp.re8n rno JoJ a,vr' ueql apruedc at1+ roJ asrom ou'ure8e dn paurn+ daql Stmrrour 1xa51

r{rlq A '27

lualor^ llprluassa sr plrom aql (c) aruapuadapur o+

'8unlI>l lo; sarlard uer-ulmJecl qlmt pagnd acer8 ruol; ualp; lsg1ced pasdel e 1 (SI) 'sasou leau /Eallnq aw'zz' at{l Jo IaaJ a{l ol Surmrw lpnoarySu'pres 1'sqlnoru rno ruoq pooJ aql

(S)

'rusnn^^J€c

u.nprr\ qe4ualod ar€ uoqn)asr"o

@Z)

rvl^le1,,, 'EV

aq1

aIISTVC sr Surznrollo; aql Jo

ptO 'Ual >IJnq) auo s,araq; (gg)

'qlrmquoJ a8els uo aru€f, raIID{ adalm.eq aq1 'p,req dn asoJ aur aprsrn Jarapmru aql aaJql-oml-auo O '+xau dqeq raqlouy 'pr€qJ ssu s Jo J€aI e w pa{oor{ ilns ^Ir€a qlaa+ alpaaulaq'1a; pue 4E aql ur peddogdW aqs larpour eql paddorp I ralel se+mmu uaJ

(s)

'aIqeIJI+sn!

q 8qilIT auos ler{l

dll^

auaql

'a8uer;o

+no luaruaspq-qns-qns e peq ddqr

'1q8rr 1no urnl +,uprp $lJnqJpoo^A. aql SursseS

(s)

(v)

aql 'I7

rnq

'auols8urppnd qlpn lnqs paruoqaoqs slrxa qloq (g) 'lq8rpre sezvr ruaql lsure8e peq ar\ aseJ aql pve aurl auoq aql 1e prnb'IryrJJaur se parnleal s€.&,!. a8uer;rxg ure.r3 pue p€C aql ruog quoq lno)JorDl aql

ol

sra.^A.sue asoor{J

pallFua

MolIoJ 1eq1 suonsanb aql uaqt /u.nxn) aurx€trAl f,qs4cru4cpoo74 uraod aql peau 'tg-0-T'-ffipsa-fr-o

"vr,o1aq

44. All of the following statements accurately describe line 4 EXCEpT

(A)

47. Whichof the following statements describing the setting of the poem are true?

The legal rhetoric of this line reminds us of the

I.

perversions of the legal system. ^listoJicglpuns (B) " Airtrght" on its legal mea_iing and its literal meaning in the context of glssing. (C) The pronouns in this line establishL,,.r, against them,, mindset. (D) The line proves that the speaker,s attitudes are

(E)

m.

described in the poem. The garden syn6ohzes nature anc thereby enriches the speaker,s allusion to Darwin. The garden makes this poem a pastoral

rV.

Poem. The specific references to the garden

II.

correct. The aural closure provided by the end rhyrne echoes the sealing up of the woodchucks, den and the closed mind portrayed in the

provide a realisfic setting in which to

Poem.

V. 45. \Atrhich of the following best describes the tone of the poem?

(A) Righteous outrage (B) Helpless sorrow (C) Ironic satue (D) lndignant protest

(Sl

(A) If only

the woodchucks had all been killed, the garden would be safe. (B) Even garden-variety violence is similar to the atrocities of the Nazis. (C) If only the woodchucks were all dead, the speaker could sleep better at night. (D) If only the gassing had killed the iaroodchucks. the speaker would never have had to con_front the violence in his nafure. (E) If only the gassing had killed the woodchucks, the speaker would not have had to see their

consider the serious issues raised bv the poem. The garden is a sy.rnbol of threatened civilization that must be protected from encroaching predators.

(A) I, II, and III (B) I,II, and IV (C) Ut and V (D) II and III (E) All of the above

Quiet triumph

46. What is the most important thematic point made in the final two lines of the poem?

The stereotypically peaceful garden is

an ironic ru-ttirrg for'the violeice

..

48. The word "Nazi,, in the final line of the poem is (A) a metaphor (B) an allusion (C) a simile (D) a paradox (E) a metonlrm 49. The phrase "beheading the carrots,, (line 12) is an example of

(A) ametonym (B) metaphor (C) personification (D) anthropomorphism (E) s)rmbolism

disgusting deaths.

50.

"Y'in this poem refers to

(A) the poet (B) the father woodchuck (C) the narrator (D) a Nazi (E) Darwin

N

CRACKING THE AP ENGTISH TITERATURE EXAM

t6lI

z

lsil

r)rr)vud

.OS

.NOIIf,ES

OCI OJ

NO OD ION OO CI]OI lnIV NOA -IIINN II NOIJf,IIS ,CIg'I]Vf,OI

SIHI NO XTIOM UNOA )JIIHJ AVI^I NOA I NOIDIIS CO CINtr

SI

AruI

lIUOdlIg HSINIJ NOA dI

dOIS

A PlrE,AI

AI

,ITI,II,I

G)

pue'il'II'I (c) IIPIrPI ())

PU€ III (s) ApueII (v)

AI

'uraod aql sazrJ€ruuns 'r.uaod aql Jo uolsnlJuoJ aql sMopPqsaJoJ

dlprruo.rr luarualelsJaprm

lI A

EI

'AI

,,'letQttrr,,

pue,,alrlJalla,, riloq u€aur

o7,,1t131t,,

pJoM aql;o dlrn8rqure aql uo suJnl

lI 'm

'3uo,rzvr

'uolsr^ urealp I€aJJns e JoJ waod aql to apn+{Frlslral arp suopu€qe 'ursqeluauruoJr^ua lnoqe 1u1od leroru 3uo.4s p sa)eur 'ruaod arp Jo lsar aql slJlperluoJ 'Strgaay sr raleads aql aJu€I€qun

. Ieuo4oura

.

pue lq8rr;o uoqsanb aql sasreJ lI 'al4erJ€u Sumsua aql seruerJ lI

lI G)

'rusrJseJ sasJopua JolEJreu ar{J

'uorsntuoc s,raleads aql sal€4suouap au{ ar{l Jo ssaussalasuas ar{I G)

(s)

'craodoleruouo sr ar4l aql Jo uoperallle aq1 (q) 'suorlJe s,ra4eads

(v)

;uraod arp Jo ezuels Ieug aql saqrrrsap lsaq SurzvrolloI arn Io qrr-qM 't9

loqurds (g)

uorsnge (q)

tmd (r) (f atlD ,,lq8p4e,, prom aql

aql to rorroq aql sazrseqdrua ,,JalIDI,, unou aq1 (3) 'su€runq Jo alor drolepard p.rnleu aql sazrseqdura ,,ada4aneq,, arulcafPe aq1 (g) 'Suqerado sr raleads arn qJrqm raprrn sapnlple IErn1Inr aq1 yo dlqerclJlue arp sazrseqdura ,,rDIMquoJ a8els uo atueJ,, ase.rqd ar6 (V)

uoiourdxo (g) roqdepur (y) e s€ suorprmJ

'I

euorprmJ uraod stql Jo au1l lsrg aql saop rvroH 'Zg

lI (a) lI (J)

aql aleJlpu saru^ril lupls aql

'11

'gg

zluarual€ls qrlq^a dq paqrrasap lsaq sr VZaqlJo uolptmJ ar{I

'Ig

SECTION Total

II

Time-2 hours

Question

1

(Suggested time--4O minutes. This question counts as one-third of the total essay score.) The passage that follows is from The Secret History by Donna Tartt (1993). This introd.uction sets the tone for the rest of the novel. Read the qas.sagg carefully. Then write a well-organized essay in which you characterize the narrator,s attitude tow-a1d Bunny's death. In_ your essay analyze the liteiary techniques that the'author has used to portray the narrator and his attitude toward the events that followed. Be suie to include specific references to the passage. '

The snow in the mountains was melting and Bunny had been dead for several weeks before we came to understand the gravity of our situation. He,d been Line dead for ten days before they found him, you know. It (5) was one of the biggest manhunts in Vermont historystate troopers, the FBI, even an army helicopter; the college closed, the dye factory in Hampden shut down, people coming from New Hampshire, upstate New York, as far away as Boston. (10) It is difficult to believe that Henry,s modest plan could have worked so well despite these unforeseen events. We hadn't intended to hide the body where it couldn't be found. In fact, we hadn,t hidden it at all but had simply left it where it fell in hopes that some (15l luckless passer-by would stumble over it before anyone even noticed he was missing. This was a tale that told itself simply and well: the loose rocks, the body at the bottom of the ravine with a break in the neck, ind the mucidy skidmarks of dug-in heels pointing the way (20) down; a hiking accident, no more, no less, and it might have been left at that, at quiet tears and a small funeial, had it not been for the snow that fell that night; it covered him without a trace, and ten days later, when the thaw finally came, the state troopers and the FCI (25) and the searchers from town all saw that they had been walking back and forth over his body until the snow about it was packed down like ice. It is difficult to believe that such an uproar took place over an act for which I was partially responsible, (30) even more difficult to believe I could have walked through it-the cameras, the universe, the black crowds sprinkled over Mount Cataract like ants in a sugar bowl-without incurring a blink of suspicion. But walking through it all was one thing; walking away, (35) unfortunately, has proved to be quite another, and though once I thought I had left that ravine forever on an April afternoon long ago, now I am not so sure. Now the searchers have departed, and life has grown (40) quiet around me, I have come to rcalize that while for

I94

II

CRA(KING THE AP ENGLISH LITERATURT EXAM

years I might have imagined myself to be somewhere else, in reality I have been there all the time; up at the t9R bf the muddy wheel-ruts in the new grass, where the sky is dark over the shivering apple biossoms and (45) the first chill of the snow that will fall that night is already in the air. Wlu, are you doing up here? Said Bunny, surprised, when he found the four of us waiting for him. \A/hy, looking for new ferns, said Henry. (50) And after we stood whispering in the underbrushone last look at the body and a last look round, no dropped keys, lost glasses, everybody got everything?-and then started single file through the woods, I took one glance back through the saplings that (55) leapt to close the path behind me. Though I remeirber the walk back and first lonely flakes of snow that came drifting through the pines, remember piling gratefully into the car and starting down the road like a family on vacation, with Henry driving clench-jawed through (60) the potholes and the rest of us leaning over the seats and talking like children, though I remember only too well the long terrible night that lay ahead and the long terrible days and nights that followed, I have only to glance over my shoulder for all those years to drop (65) away and I see it behind me agairy the ravine, rising all green and black tfuough the saplings, a picture tl-rat will never leave me. I suppose at one time in my life I might have had any number of stories, but now there is no other. This is Q0) the only story I will ever be able to tell.

96ll

Z

rslr l)tDvud

s8ururumt .a .e .ralem Jo sse13 e dlprder Tuerp

puv

.o1ods

all

,,Zalnru aq &raq[ Jo aJro^ erTl lleqs u€q+ pealsul parp daql TqnD ol dols lou p1p daqf .ralq8nels Surreor aq+ o+ srrorl aTrI par{snr oryr puap dddeq rlorar{ asaqt ueql lW -ln€aq aroru aq plnor +eqrr,r' flneaq;o 4e1 dqrt urn8 dq qso8 dq aa8 dq o8urf dq drro8 dq aureu snorrol8 rnod urrelJce suos rtql

OI)

Lf,""T,H,,, "",Tff,i:1"TJifr;_1::ff oB pue aurof, salJn+ual Jo sr+, [,4rmot autT

dur d1.rea s,u/vr€p aq1 dq aes nod uBJ des qo rluoJ os pu€ ,sur.r8pd;o puel nod a,ro1 I E)rJaur€ po8 asrnoc yo ol lxau,,

-IIa$

'rusrlor4ed;o lrlall s,tuaod aql seqrurexa 1eq1 dessa paznre8ro € alurvr'uoBcrp pue'auol'a1d1s se s+ueuala dreralq qrns Suuaprsuo3 dgryarec ruaod 3qmo11oy pLnU "W

('arocs dessa p1o1 aql Jo pJ.n{+-auo sB slunof, uopsanb 7

uo4san[

sryr 'salnurur O7_aury palsa88ng)

Question 3 (Suggested time--4O minutes. This question counts as one-third. of the total essay score.)

Lr some works of literature, mothers or the concept of motherhoodplay central roles. Choose a novel or play of literary merit and write a well-organized essay in wirich you discuss tire matemal interaction between two characters and how that relationship relates to a larger theme represented by the work. You may select a work from the list below, or you may choose to write about another work of comparable literary

merit.

ADoII's House The

The

Awakening

loy Luck Club

Medea Mr s. W arr en's

As I Lay Dying Beloaed Black Rain Bleak House The Color Purple

P rofes

sion

ARoomatithaView Pedro Paramo Pride and Preiudice The Scarlet Letter The Seagull Sons and Looers Sophie's Choice The Sound and the Furu The Stranger To the Lighthouse

Daniel Deronda Dombey and Son

FifthBusiness The Glass Menagerie

Hamlet

STOP

ENDOFSECTIONTI IF YOU FINISH BEFORE TTME IS CALLED, YOU MAY CHECK YOUR WORK ON THIS SECTION.

195

I

CRACKING THE AP EllGttSH |_TTERATURE

tXAil

QUEST|0NS

l-15

"An Invective Against Enemies of Poetry" is excerpted from pierce penniless, His supptication to the Deuit, bv Engtishiatirist Nu'r" irsoz-roor j. a lou;";t** lhgrnas Penniless in about 1592' Although uy,lorrrnuo,,rr* much oi his work *lr"rJ be considered reactionar y bigatry, Nashe is admired for his enerletic u"a ""* ,utuiirr.t;.m'ol;ilnding prose style. In the passage t;::il?,fff

ilffi;ffiil

tr::TJ'"ffi yJlii:ff 1' D

ffi ,:Xtl"'i"-r"*ii";rcon-tr'ibutio"io,oJ",v,especiary

"Eloquence" is the obvious answer here, since it runs contrary to the meaning of the passage' (And you remembered you were looking ror ** .;.,i rary, right?This is one of those "except" questions.) All the oth", u.r*"rs straightforward acc-usations in the text, except for (E)' which is a little obscure. But when _are ,,quarter Nashe

tuirc uuo.rt the preachers, sermon"'he means a sermon given once a quartel, only four trmes a year. The implication is really do-n't work very huta,'"nrii. po.is, as the r,.*t

pur'gr'uph goes on

lll-fr:fl..achers

2.

.

be tle pranet of depression and groom, and ffiXT:j,:*:1",e;1s,,3i:l y:,":T:shj l"

ffifi?:#:ffi :ii""ff ;:, "o-?:0"':-T:",gll?lyr.."*,J.iffi ;!*;i*H*'r'?ffffi Irrrrc ulan lt ls liilL'n1::"#:.::1lyt"::':i:gltri:Ff that saturn is;;i;;;,; jump to ,,astr 'fffr'#ix) :::;Tl"t: iX f:3*1!"-:;o1tlu

i;#ll*:::f

::::13'.'^",,i;91:":',nrlllly;i;:$d"1il*iJT?lt?.#'-*;:T:#,:';lJ jg;;a;ffi

ffiff ;:il:i#:J'"; l#*,yfi:':i"y::::):"*g::iri:it"i up,"u.hu..(E),,opt?mi,,t.;;i,#;i#ii::i#,: ffItJrff

ff the required definition. of

3' D

+.

":?lf#::'*tiJnof

unlike thelazy,nlisr.alizing

preachers, poets must have new material all the time or the public won't buy their books. (A) is wrong because the various professions presented in these lines are only implied comparison, io, u poet's .u*., not a_list of moonlighting poets. (B) is wrong because Nashe's auiience do"1t:jd;;;; poers; they only reject bad poets who present stale, recycled material.,(C) and @f are ffiru,ury irr.i"uu'itoih" purrug", but might tempt those who think in terms of ,r,urt competition. "tpiu.l A Nashe is comparing poets to fishmongers, who must constantly keep their product fresh for the marketplace' He is continuing his"thought about poets being superior to plagiarizing preachers. You courd make argurients in gupport of (B), the idea.that poetry hasslippery meanings' but Nashe thaq he;s del."d#;;;;ry (C) and (D) are also interesting ideas' but they :-lt-lr*l.t are not in thii passage, and are in"fact contradicted in other portions of the reading, which praise the special citural just ridiculous and completely el contradicts the meaninj;f ,n.

"r.uni"g;;iil;;;J*"oiilr. passage.

5.

C

*

Nashe is asserting the p,lrity and beauty. of the English language, and argues that London, as the seat of literary c,rltute, influences i; ail over the country, like the streams of a fountain spreading outbeyond the iountainit*r. ri..ur*., p""o r*prove English in London havei national influence. Answetrlor, (B), and (E) are just incorrect associations with.the watery imagery fountains

h"*.Effiil ;i"A

*i"

of

ffi:XT llffLt:}]*

I98 N

follr

lempting because it includes

is the souice of luliure, but does not'include the-importar,t conc"pt or

(RACKING THE AP ENGTISH I.ITERATURE IXAM

661

I

SN0llVNVldXl 0NV

SUI/trtsNV :Z

IStI t)tI)VUd

'luexa aql aroJaq Tooq sry] aq+ ralo pear ol arns aq ppoqs nod'surral asaql ,vrou) r,uop nod I 1,,;i.p.ro^;o qJo noA iuooni 9,,) uraod aql Jeaq l,ueJ +Eql Suqlauos Jo auoaluos sassaJppe dlna,rp uraod e uaq1vr sr (g) aqdo4sode uy 'asuas .radaap aruos ul an4 aq o+ lno surnl ]nq uorlrrpe4uoJ e a{rT spunos 1eq1 Suqlauos sr (3) xopered v'spunos pDads aleru l,uop sarra;pd pue suad 1nq'(,,raqlqs,, ^?a) are daql leqm arrT prmos sprom uar{,vr sr (g) eradoleurouo .lp)rror{delaur lou fl srql 1nq'raqloue Jo sural ur paqrrJsap sr 8try{l auo uaql\{ sl ,(v) roqdelaur y .8uorm 1sn[ are sJamsue rar{io aql ilV 'au]l e ur dypaleadar punos }ueuosuoJ lel}r-q aues aq1 Sursn suearu qJr-q/vr'uoqe.ralqp 1o aldruexa ue sr ,,qsqod ol uad rnod;o sarra;1d aql,, aserqd aql

u dressolS

C'OI

'8uryr1a4s pue SurpearJalo aluos sarrnbar,,sla8ue,, ayqrr'uosrreduor +JaJrp e s srad.uel leql aas o] alqe aq p|nor{s nod'leql dq pasn;uoo araa,r nod Jr ua^g .arard ptrr4es E qrns Jo a|pplru aql ur iou dpnelrar 'qa8ue ol sallaswaql a;eduror dleraua8 1,uop slaod pagaJuoJ

uala puv'dlarms Jo sraquaru se qaod lou'1daruo3 oq1 :drqao4 lnoqe 8uq1el sr arualuas lng 'saserqd aql Suqeads qa8ue aq} are slaod 1eq1 dldurr lq8pu qJr-qm ,(99 aqD ,,qa8ue ;o aserqd dral aq+,, aq ol prcs sr d4aod asnpJaq dlorrl aFlll p sr ,qa8ue ,(g) 'srJqder8ou -orqJ ueql Jailaq aJe slaod se 1sn[ uaursumo] uprn Ja+aq are sradrvrel :uoseal atu€s aq] JoJ Suorm sr (q) 'parroJq sl (J) os 'uaql a{{ tou's.raqderSouorqr ueq} railaq aq o} pres are spod pue'suerrolsrq plol aJe s.raqderSouor.lf, 'lnoqe alr.rzvr sraqderSouorqr Surqlaiuos 1sn[ aru daql laraq Surqldue o1 pareduro) uala ]ou are srodeu asneJaq Suorrvr sr (gj .uerrolsrq ueql drolsrq rnod a1rr,u laod e aleq ol ragaq $ 1r ,os 1sn[ prre'(gg-lg satnf uerusu.,u.o1 _E 3ura11n1s e ueq+ ,,asneJ s,auo peald o1 rar{rvre1 lue8ala up alpq ol,, Ja}+aq sr 1i sdes aqspN

V'6

'luelalarrr 1sn[* (y) 's1aod Jo lsrl sn0 dq pauqaqzvrralo 8ur.pa8 ara1vr nod;r nod asnluoJ o+ araql sr (q) .spueqsnq a^€q sa^r.lvr asnpJaq'(l) Sulsooqc o1u nod ra)Jns l{8lu aJI.{{ Jo uorJou aqt,palzzeqdran 3uq1e3 ararvr nod I 'a8essed sF{l ul pauoquaru sr aqs pue 'q}eg tuo,r1 s,ar{s asneJaq'qleg Jo aJIM aql palleJ raJner{J uI ra}Jurer{J snoweJ e sr aJaql asneJaq Sursn;uoc aq lq8rur (g) ramsuy 'ua+snv aue{ ut oslE lnq rarner{J ur dluo lou dn sMoqs umo} ar{} 1nou1 o1 papadxa 1sn[ are nod luql uoqsanb a8payvroul praua8 e $ srqJ 'pue18ug ur eds pue u^ro+ luarJue ue $ r0eg

g'8

'qleg Jo aJIM aq] uo {rn}s 1oB pue a8essed srql .aueu Surpear aFq,rt ralo paze1? sada asoqrvr aldoad roy s1 (g) 'plalJ ual Jo lno lq8re.4s sr (q) aleuqel aql dq pa1sa88ns lnq paJJo)ur dldrurs sr (3) '(,,aureu uad,, suearu 11) sueaur awnld ap M7u l/Jlqii' lvlouT or{lvr aldoad asnJuo) ir{8rur (g) '(oogt-r7g1) sepeg np a}snFs ap arunellrnD se depol u1lroDl dFergure; aroru dgenlre '1aod qtuau e sr aq nod q1a1 snusnles arueu arD ^ Surzvrollo; dltcarrp aserqd IeJqar{luared aq1 lprervr,ropq8re4s &lard ,{y1en1re sr uoqsanb sHI

V 'l

ler{+

'sau{ parJrrads aql;o Suuearu aql urory plaqe reJ Buqp8 s,}eql }ng 'are1d lsrr; aqq ur auodrala lnoqe 3u1u*r are slaod aw dq,rt ur dn lq8ner 1aB pue {u1lua^o oqm asoql roj ldarxa 'a8Bssed aq] r{llm op oi 8urq1,,(ue aleq dlear l,usaop (g) .1noqe ,(rrom o1 suoqelndal qlIM aldoad;o ,{ro8alet Jaqloue se idarxa slsrcq}e ur pa}saJa}ur lou $ aqseN aql Jou poD JoI Jar{llau aJeJ asoql,, ol aJuaJaJaJ 1nq ,,'lrrap aq} uo snJoJ or{n aldoad +pr{+ >1cr41q8rur (q) '8uurydn sr d4aod leql papl praua8 ar{l }ou'suorlelndar rraql roJ slpnphrpur Jo suraJuor puosrad arp dlleJIIIJads sr a8essed sgl uI uraJuor ar{l tnq'd4aod dq parrdsrn are srarplos leql uo4uaru saop aSessed aql asneraq 3u4dua1 s1 (3) (rpJ seM aq {ultg t,up1p sarrerodrualuo) snJ os'auqalT srq Surrnp JaJapuBIs e palaprsuoJ uaryo se.rr ar{seN,+q8noql aleq dew aq leq,vr elrdsap puy) 'saut asaql rn ssaurreJ lnoqe pres sr 8tlq1ou lnq,(g) qum aar8e 1q3nu aqseN 'suoqelndar Jlaql JoJ rea; dq pue 'aureqs rorror{ Jraq} Jo aruey .roy Bur -3uo1rraql dq'uragl

Jo

lnoq'

pr€s sr r'q./vr dq pale,rqoru are aldoad ,nroi{ aqrrrsap gt-6€

saq-I

v

"',#?:'#:iff ifi l;:;'5:TffiTiil:;1x;Hffi 12'

A Choices (A)' (c)' (D)' and (E) are all defenses respect' leaving (B)' which just is

repoffi

A This is a pure definition question; you need

13'

*aightrorwardsrammarquestion

that Nashe uses for why poehy deserves *ni.r, p-"pil;;;i.pr in awe by poets,pens.

on

to know what a tautology unnecessary repetition,so (A) is. A tautorogy is is the right urry:r..(u), and so seem to fit'but do sound like criticisms, not paraphra'se the cited'lin"lif;);;t; says that the preachers but sermo"s u'e,tof by definition ineloquent, as siecified in l,?J;.**""s' the sentence

(q,;;6iail

E

14'

The selection specifically mentions all other options as functions of poetry: (A) in tnes 54_Jz; (C) is in 1i"., is in line 40; i, function of poetry but is hne 30. (E) is not a mentioned in the selection "u, u ruil ir. or lor-g preachers. Thf is a hard question asking for general knowredge. you courd ha rearized that peopre necessirv.or poerry have led youlo thoo" the correct *r*u.Ipluto. (Actualry, irili"'a..ia.s that poets are-n,t reallv necessarv in his RE.ubtic.) (B) is

rs.

D

4&2;r"jtol

;ffiffi;om

n"":,r:1:F;""t"ili: N";h; (;j

#:r*:;$rffi,::1;fn*r

;:f;i,:T:1,il,:*'jilr'

ffi" ;r,#;i,ffi[:tion.

rn*uonua i,,ihe te*t. Milton

G);th.

sidney (B) and ramous British poer

SUEST|0NS t5_28

john Donn

*,'",,*niffi.i"|r;:lifff xiiif,iil:,;,iff iiliii:!r,*,,:ry::#ff HX";H"'ff ptoru*lq"b'"L9,;;#.;;""toriously

subjects both sacreJ utta

figurative language, which is why he tends,rh"* This poem, "A valedictio"' ori,vu.pir;;

]lill:luf;1?if:,',:ff 16'

C

shirts

,o

airn.rrtl".ause

on English

frequentty. .;d;s:I"q:;o*;.*,"*u*, *rrated from one,s

oipe*pu.tiu",

i?

phys with

ihe;J,d";;,";i

%H;

of his ingeniously rover.

presence and absence,

rf vou are finding ir difficurl:o^r:1{:.tr:lv yh:t ihir-pg.* is about, vou can use poE ro answer this question' The easiest one to eliminate is @j, ing here for a pleasure .rt^.-:-S:_: is far too much cry_ ,:Td apocalyptic, so (D) might tempting' but you should sound have realized that,ti, o.u love poem and that the world imagerv is meraphorical end of the metaphorical death' so (B) is is probably arso a wrong' That leave! (A) and (q. ro te' if

;;Fi *;:

becau#;

lt*:J,."*l *"" tGffi;;it ill;;; the reration_ l"i,tu,,.., but the rou*lu," derinitery being :$m: nruU"it:X":S**'.1* IT.

D

youshould know the defin

Exam'ikes''"*ffi;;rl:i, j#iTir-^{{r,.::Lff T:'f ffi fl IT:',:rT::,:y,. v"I'r'n""ia;;;;;;d;ft this as a metaphysi_

simile that occurs in a meiaphvrl.riil;. cal poem' since it is by JohnDtr,r,e. br,ect Note that "metaphysicul" h.r. rtur rittr" to of meaning' metaptr^ttjT*..",u tear becomes'tre an entire world once once someone pastes images

iJ,iJir.ru.y -ou"'n#t

i"

overview in chapter 2. ,"rin il"i.r*pi;;^""1n'r.n ro do with depth isthe coirlarisii oi,"u* to

+iull#

grobes-each

j* ;;;;; bails become globes of the continents on them: 7";;t#; been rempted to answer (C)' recognizing tiat this is r;rfi;;;rical, but in this instance the lines it reflects his beioved,

are

2(l(l

;

CRACKII{G THE AP EI,IGI.ISH I.ITTRATURE IXAM

l0U

I

SN0lMVtdXt 0t{V SUlrltSt{V :Z lsll t)tI)VUd

nod pue'uraod aJqua aql lnoq8no.rql patnelsns uaaq spq dra8erur punor aql asneJaq Eq dlsnovrqo (l) 'sapp pw ,eas aq1 ,uooru aql uaam+aq uoqJarruo) snorlqo ue sr araql asneraq aql sllJ ($ 'srarvrzue aw uI llJ +,usalp lpr{,vr roJ 8q1oo1 ,rn nod os ,aiaq rfunrrorrn ^nraod Suotm aq dpuapl o1 8uroo1 a.re nod 1d1ryarur uoqianb iq1'pn", ol raqwauau .araq tno nod dlaq uer uorleu.ruIlH Jo ssaJord prrsdqdepur *ouurnroc aql qlrrvr dlrrer io {Q'ft+aod lIurBJ auros pue ruaod aq1;o uoqeprdralq qloq sarmbar lr asnpraq uoqsanb ,(rl;li ; q sF{l

o

.02

'8uorm sr (g) os ,asnep luapuadap aql dq peqryoru lou eJe lnq ,,ueluTro1vr,, pafqns aq1;o spa[qo aru quauquor palsr1 aq1 .8uo.rrrr. sr (61; os-,,qJrum,, fq pngroo*'e*rq .buorrra lou sr pahns aql lnq'asnelt luapuadapur sHl Jo loahns aql st uur4ro; s 131 ".I,l os',rea1pazr_oqdepru e $ ll u€q,l ralel sarq ezuels sql uI dn,r,roqs ua1a lou I€JaAas Iqlm saop ,,plroM,, 'SuoJzvr sr (y) os 'Srmndue olul waql a{€ru lou saop lnq-sardor aql ol sJaJar aq ldoqs sFI u.r ,,dq qleq,, uerDIJoM aql slaporu ale ,,sardoJ,, aql 'qapow tuory pardbr rrn .nqo1d ,aro;araq1 .doqslrorvr aql q8noqllv 'sIIPq prmor aql ol raJal lmlu ,,qJnl1vr pw ,, ,,lE:!{L,, rfloq .uer1nlrollir sp.l uI Tuplq arc +pql sileq plmoJ aql aql Jo lno saqoy8 Suxeru $ upru>lrom arll sl ,/a{Bru,,;o pahns aqL/?Teru t, qtalaql;o pafqo aql fl ,,lprll ,,.s1 ,,+etq+,,Jalaleqa ol sJaJaJ //rprq.{vt,, os ,,'+erll,,3urr{rrporr asnBIJ luapuadap e su€aq,,9JI{M,, ,uraod sr-ql uI arn+Jn4s aJualuas aql lno uos ue) nod qarvr rvroq aas o1 uoqsanb reuurerg e dlear q s1l[

g

.ZZ

3

.17

IIV g

.02

-

(s)a1u'.n4rs.s,a^oraq+uaa'+aq,.tr"",:I:ffiT"1',#$ll;:f:,:lillJ::i:,.{ililJtr q (d 'urao{ aql lnoq8no.rql pauplsns -

Suraq uuql raqler 'ezvels puoras aql ur luaururord dluo dpa.r sr dra8eun aqo13 aql 'a.rour.raqlrng 'ruaod aqiyo Burpear'apldruorur ue sr (g) Bu -sootp os 'sreal lo sa.laqds dup aql tmlll,lr uraql Swureluor dq saJuelsrp lsEA aql aJnpar ol sI uraod aql yo lurod aloqm aql lnq'sacuelsrp lspl pue saqo18 ere arr,p ,srrnr"q,r,{gp n".{ st (g) 'ssaussaladoqgo asuas E araql $ Jou'ssaussalllrorvr yo ,{.ra8eur due irrrnr"q 1,usr

(y) 'aldruexe up q sr-q+ qJr-qm 1o d4aod alol rn relndod dp'adsa"r.q1 ,rprr, "pn* saprynb asaql 4Fuap pue ssaualoqm paFad e+EJrp,o pue pua ou a^€q salrrrJ "ro;irrql 'adeqs pa;rad 1{l se olrrlJ aql sla,rdralur d4aod drnlua>qluaalualas Jo-lerroleur prroprd ro-dqderSouoJl atp asneJaq'dra8eun alJrrJ rneluoc suraod prrsiqdelatu f**,X*1a 8uo.'vr

s1

,,'alaqds,, pJo,lr aql pue uooru aqt

seq ezuPls PJFD atf,],'splrolvl pue saqop s€q earels puoJas aq1 dem e ur drueu$a.rd pue 'llruJ pue stnoJ seq ezuels lsrrl aq1 .ssaupunor yo sa$erur rneluoJ spzu€ls aql

uala

Jo

-rorur aplrrur (a)

pue'(d'b)

'parrorur aroJalaql $ pup'arroqc lsrg aql *qs {tiTl}l1td; lrar.ror ale_ saJroqJ puoJas pue lsJq aql ,arolaraql .sJalen Jaqp ol 1o,r,rrffi ol $ sarq snonard aql tuory Sumuguoc q leql uosrledruor aw pue,sauq oru; asaq] rr paJualeJal dlerunads lou arp daql'uaod aql rn pauoquaru are *ei pne rn"r q3no.11[Vl8rrt -qlolJ lnoqe areq 3rry11ou sI eJaqilaJpJ Jaq uo ,,uro/vt,, sreal s,pa^olaq aql osle aJe daql pue 'paropq srq;o a8eun aql ,,reaw,, Wrqaa ,srea1 s,raleads aq1 ale gI pue,I saql ul sJeal at{I

g

.6I

'(3) pos s,pod arp suonuaur uraod sFIl ul Stmnou puv'srure ou serl ($ pFo,,vr aqJ'uooru aql (v) 10u dlsnor,r,qo g .**1ro^ aq+'(d ro (g) raqlra ol saJror{r rnod smorreu qJrqm ,(tZ aql) flure setl dlqua passarppe a{l asneJaq sul panlear a^pq ppoqs no^ 'pa^olaq aql o+ ssaJppe up sr ,,uooru ueql aroru o,,

iI

.gI

ul

ua$

se

f

aql alerrpur ot (?I aqD ,,rllop os,, Jo asn aql aloN :alnurs e lnq,roqde-"*;';;"*jJriT;

should remember that you have already been asked about round imagery. Remember to keep your answers consistent across questions. (E) probably fits because the poem certainly does describe.unhappy feelings. Thatleaves *itil (A) and (D); you must-ciloose between a goddess and the man in the moon. There is"r no explicit refermce to either a goddess or the man in the moory but at least the idea of a goddess seems flattering to tn. u.toved and fits

24'

D

25'

A

26.

B

27'

C

the poem better. Therefore, (D) is the least fikely association and the correct answer.

"Diverse" here just means "diffetent";the phrase means "a different place.,, Heaven (A) and hell (B) have no place in the poem at all. Europe (C) is listed as a contnent in the poem, but not specified as a destination. The ground (E) is jusi an answer for the truly Jesperate. Tears are found throughout all three stanzgl,.although you have to look closely at the final stanza to find the idea in the word "weep" (line zt)."Glfbes (B) ur" oo1f i1tt*,".ora stanza' coins (C) are only in the first stanza. The moon (D) is only i" tri. tr,i.a stanza. The ocean (E) is in the second and third stanzas. ocean is almost suggested by the mention of shores in the first stanza,but it is not as r-fu"g a suggestion as fr"at of ,,i""p;, for tears in the third stanza, so tears (A) remains the bestinsweli

Line 4 is "And by this mintage they are ,,They,, gomgthing worth.,, refers back to ,,my tears,, in line 2' The tears are said to be coined by the betved's face (line 3), and the coin metaphor is carried forward into the use of "rnintage" here; ttre process of being coined is the mintage that makes the tears worth gomethin{. The tears reflect her face thi *uy that coins show a ruler's.face. By showing the beloved's face, the tears become valuable, like coins. Therefore, (B) is the best paraphrase. (A) is conhary to the sense of the poem, because it says the beloved is- worthless. (C) does not reflect.the meaning of the line, some readers might be confused if they don't know what ,,mintage,, "itn""gh because nieans. (D) is tempti;g, it sounds nice and fits with the meaning ofthe poem,"frt it overstates-it goes beyond what the line means. ut" ,uppor"d to ;estate, not extend. (E) is b;th;ague and faraphrases sort of New Age, and is not the point.

The speaker asks the lover to "forbearf To teach the sea what it may do,, (lines 21-22) and that she not let.the wind'Example lind" (line 24) in her behavior. ri. i;,;yi"; rhat the natural elements watghing and learning from her, copying her behavior, ai idea that is carried forward-a1e ,h: description of iis lover ur tr"rl influencing the tides. One would think, in the face of such great love, that she would have the po*"rio ur"ak his heart (A), but that is not mentioned in the specified lines. He does mention dying in these lines (B), but mostly in the context of her power over the elements; he warns her against teaching the oceans and winds how to kill him, not against killing him herself. The lines assume that she has the power to restrain her grief (o), b"ut just tell hir to do so; they do not emphasize this power the way thgl.do her p6wer over nature. Nowhere in the poem is there anything mentioned about the right to take other lovers (E).

ft:-

28'

202

S

A

-*"

The speaker flatters.the beloved by exaggerating her powers over natural elements, by declaring the inestimable value of even theleflection, oi h., image-in other words, through hyperbole.

CRACI(ING THE AP ENGLISH I,ITTRATURE EXAM

toz

I

Sll0llvt'MdXl 0NV SULT SNV :Z ISII l)tI)VUd

'ramsu€ alqlssod dp9 q (y)'aro;ararlJ'aslpJ sr luaualqs aql-$lrr4 due auodue qreal ",tl ol3qd4 sI auo o5:Stmnduu uo lq8q Iear ou spaqs leql acelduoururoelueddrg e dprau sr

A 'an4 alotaJaql $ pu€ 'a41 urvro Jaq ol stro4Jeal s,€slno-I dq pue aldoadsumol aql Jo suo4 -Jear atn dq papoddns pue a8essed aql dq palsa88ns auaql € sl AI 'ueruog Juarrup u€ se^ resaeJ l{3noq1 pue a8essed aql paurugs oq t aldoad asnJuor o1 araql sn11 .a8essed srul rr IaruJ s dlanos 1eq1 uorlsaSSns ou sr araql'1uau4ea4 s,Jesae3 yo arrorddesip nod ua1a Jr

asneJaq'a8essed aq+ of luplalal lou sr lnq'eapr Suqsaralu ue $ .anJ+ aloJaJaq+ sr pug 11 'a8essed aql Jo €apl rneru aql sassardxa I urall 'dlaleredas luarualels r{Jea arnru€xa ueJ aM

V'I€

'eJne arnlnJsetu aloru s,aof pue saIlIAIpe s,Eslnol uaamlaq lse4uoJ qr dq uorlelardralur srql slsaSSns a8essed aqJ 'sallllrlJe aurunual lleuourpe4 aleJrpw ol ursrJqrJJ lsrunual gr :1g; pasn dlaprrvr rural e sr qJrqa ,,'araqds lpJnllnJ au.n4ruaJ,, aserqd aq1 sasn (g) 1nq1 ,pp salPal srq|uMol aql Jo sanlel paq8rq aq+ +ou dlqeqord are daql 'saqrlule al4onpord-ssa1 raq Jo dueru dols raq a{uur IIIM pueqsnq pue rvtefur-raqpru Jaq leql saledrcque rqr rsnnrrq lnq's1se1 Puoqlpe4 dueru u1 pa8e8ua fl esrnoT asn€raq'Suqdural s1 (g) 'ptpl due;o snruad e

$ aqs leril

lsaSSns o1 3rry11ou sr araq+ 'asnoq.raq Suruealc agq,t,r ,,lsqre ue Jo usersnqlua .ttt dgeluaru $ esrnoT leril lsaShns o1 Etmpou sI aJarp asneJaq 8uo,rm sr (3) '1,uare sarlrlr+Je dluapleru s,esrnoT leqrvr'dlasnard sI qJrqM'lenxas dlensn aJu sassappo8 rraql pue sFn+rJ lsalJetl'saprsag 'FnllJ +salJeq

aql lsourlB,, seq esrnoT qSnoqlp 'asnelaq Suorrur sl

(d

puoseas due aqorrtn ol ruaas l,usaop leql pu€ 'sqraq;o Sugpsrp s,esmo-I sr uoqf,alas sHl ul Supurey ol aruaraJar u ot asolr sauor teql8ulw dpo aq1 :snolnrrprr lsourp sr (y) 'uorlJalas aql;o Surpear lryareJ e pue gocl qSnorql uonsanb srw ramsue unr noj,*n3+

s

'00

'aJrT parrreu gul Jo saqlnJrJJrp aql -pre8a.r ruopsrlvr lEuoqualuor aql uI salartaq aqs ,luegodun aloru lnq llaqlour s,ao{ pue ,ao{ 'J€sae) lnoqP plol $ aqs ]eqm salalTaq aqs se JeJosw +ua)ouut osle sr aqs pue !r srvroul pue

'parallaqs dluregar

$ esmo'I'(d sl rar$up alqeldarre dpar dluo aql,arolaraq; .1am dialu

-raqlTap sI rolleqaq joq uI 3rry11ou puu '8op aql sa^ol osle Esrno1 1nq'resae3 ro; dqledruds mo J^o asneJaq padde oqe lq8rur (g) ,{em qt ur'q8noua JqsrTear dlurepar sr a;u ratla$ueqo lI,!' a8erlreu Suruordn Jaq moq;o uorlelduraluoJ Jaq 1nq,qs11oo; uaru lnoqe seapr s,ESrno.I pqt +q8r-ul no^ 'lg dlear l,usaop ,,r{suooJ,, pue €qlqruar} JoJ suoseal Jo slol alp araql +nq .'sa1qura4,, psmol Tt.{ ur a8essed aq} Jo pua aql ol alelal ol sruaas ,,paua1q81q,, asneJaq €urn1e osle $ (3) '8u1p_earralo aq plnom lpql lnq,qspgrlr leq,/v\auos are uaur lnoqe suoq ,alreu -daruorerd rarl l'rfl an8re lq8pr no^'rlslpF{r BunDdue Br4op uaoqs +ou $ aqs lnq il esmol asneJaq'lq8g dlped sr (g) rvreyu-Jar.&oru alqnJ Jaq lnoqe sreal raq rpr.u psrnoT lou $ esmo.I _ pasnJuoo nod aseJ uI alaql q u,lvroJtfi seM l€ql 1nq'Suuaauuop Jo Janrq.uaruom :3uo.rzra, dlsnorlqo sr (v) 'goa asn u€J nod qrrum ur uoqsanb e osp sr sHl lng +noqe saddolarals relndod dueu o1 qeadde esmo1 yo lre4.rod alp asneraq notrranb d1Jr4 E sr sn{J O.'62 'Ile ra$e paIJJ€ru pB o11uem_l,usaop dlpar aqs lerp luaula8e8ua Suol drarr e rarye sappap oq6 rrEruo/vl € +noqe '169I uI paqqlqnd ,,'!rnN puelSug llaN v,, d,ro1s goqs aql ruory sr uortralas srql '8uqum raq ruo{ s8rmrrea raq q8norql aruapuadapur JnuouoJa a^arqJe ol alge a1u4 Jaq Jo uatuolvr l\,ral aql Jo auo osF spllt ueruaaJc .aurq Jaq Jo sJalrJM puot8a-r rn Fnsmm rpdap pcrSoloqodsd e rago srapereqJ Jaq lnq'Ja1u^rvr p.rot8ar pue ptrear dpna luepodun uE se palou dle.raua8 $ aqs 'sJlssep dreralq Jo spuarq qlrrvr suorssnJsrp rarl dq pacuangru4 dpuno;ord 1s_our lnq'palernpa-1am spm aqs 'arn+lm arussardar pue puoqrpe4 e uaruom uo slJoJJa Jo prrSoloqrdsd aql pade4rod leql uollrlJ alorm aqs'lppe ue sV'ploqasnoq Jnor8lar d1pr4s pue paqsl -.razrodrur ue ur dn lq8norq s€rvr orl 4, ralrmr puelSull rvraN e ser'r (6961-7991) ueruJarg r"nftfn .g irnru

6t-6t sNotls]n0

32.

C

There was for many centuries a well-regarded theory that said personalities were influenced by "humors" in body: Warm humors in the body caused u.g.y or passionate personthe alities, while cold humors caused unemotional or calm personalities. These humors were affected by diet and by environment. This is where you get the stereotype of the hot Latin lover who lives in a hopical regory eats spicy food, andls given to fits bf violent temper. It is also where we qet the idea of people being "in a good humor." The bitten neighb^or is described as "choleric," which is another reference to the theory of humors. Louisa is deliberately feeding her dog bland food to discourage any furthei attacks on the neighbors. She is not poor or she would not be able to have such nice things in her house, so (d) is false. The passage does speak metaphorically of Caesar's impriionment, but Louisa is not deliberately punishing her dog, certainly not for a decade, so p; is wrong. There are many references to hermits and nuns in this story, but dogs do not practice cet6acy as a religious practice, so (D) is wrong. (E) is wrong just from context, because the passage mentions that other kitchens give bones to dogs.

33'

B

"Redolent" means "smelling of," andbecause purity doesn't have a smell, its use here must be metaphorical. Irony (A) is tempting, but there is nothing opposed or contradictory here to indicate irony. There is no simile (C) because there is no diiect comparison of two things. Because there is no contradiction, there cannot be an oxymoron (D). Niothing is alluded tJ, so (E) is wrong. If you do not know these terms, study the glossary in this b"ook.

34.

A

The story of Caesar is a gentle satire on the minidramas of small-town life, which finds excitement in the vicious reputation of an old dog. Clues to the satirical tone are the many overwritten references to sin and danger, and especially Louisa's vision of Caesar on a rampage through the town. Freeman's treatment of this passage is too humorous to be either indignant (B) or pensive (C). Because it is clear to the reader, and even to ]oe, that Caesar isn't really dangerous, there is nothing foreboding (D) or menacing (E) in this passage either. The selection does say that Louisa feels many "forebodings of disturbance," includiig rvotries about Caesar, but those are Louisa's feelings rather than the tone of the passage, iuhi.h indicates the attitude of the author.

35.

E

"Sanguinary" means bloody, both in the sense of containing blood and of liking blood. (D), vegetarian, is obviously wrong because it contradicts the meaning of the sentence. The other answers draw on ideas raised in previous questions on this piece; remember to keep your answers consistent. We have already established that financial concerns do not dominate Caesar's diet, so (A) is wrong. (B) and (C) drag in the ideas of masculine and feminine traits that predominate in this selection, but they really have nothing to do with what the dog eats.

35.

C

This question testsfrow well you read Louisa's character, because the entire passage is about her attitude toward gender relafions. Her meditations on the disorder her future husband will bring to her house and on the impending danger of Caesar's release are best summed

up in (C). Louisa is living in a society that believes men and women belong together (A), but her worries show that she is not entirely convinced of this. On the othei hind, she is not explicitly rejecting marriage, so (B) is not the right answer either. (D) introduces the idea of wildness that you might have associated with Caesar, but it is important to note that Louisa does not believe she has tamed Caesar, nor does she think she wiil have any influence over her husband, so (D) is wrong. Her belief that joe's decision to release Caesjr will prove disastrous shows that she does not think men are more intelligent than women, so (l) is also wrong. 204

J

(RACKII{G THE AP TNGTISH LITERATURT TXAM

g0z

I

sll0tlvNv'tdxl 0Nv suil sNv :z

Isil t)[]vud

'(6) reguns dleuuassa sr aJualorl IIe l€ql lugod s,tnum) aas ol nod peal pFoqs aplr -oua8 rze111 qlrm qJnq)poorvr 8uryq yo uosrreduror,{ral aq1 luaod aql rn auol ol dlmqtsuas e sannbar srw ftrzuSorar q8noqlp's;a11aq snorSgar ol relluns almb are sJallaq prullod 1eq1 '(f,)Jo rfin4 aql saleqsuoruap pzuels p4ql aql'(g) alermuga ol aleilsaq plnoM nod'aro;araq1 lursrurrvrreq IerJoS pue srzpN uaamlaq suoqJauuoJ uo JealJ +ou aJe nod p uana'tnm,req o1 aJuaraJar aq+ aJrpu p1nor{s nod'aswra4t1 '(y) alermuqa o} alellsarl plnom pue uraod aq} 14 JIJolarlJ slr.{l }o aruasard aq1 azruSoral p[no) nod'uorletmrua]xa rnurJal ]o JrrolaqJ IzeN aql lnoqe ,l.ornl lou prp nod ua^fl 'anq alp sluaualels raqlo aql lo 11y'uraod aql to asuas eJqua arn slJrpp4uo) (g) 'pel uI ramflre paJJoJ aql pu€ 'aqe; sr (g) 'aroyaraql 'qsad uapre8 Sugutmuqa se qJns 'r(ressarau se paqqsnf uago 8uryu uana '8tngq IIE suuapuoc uraod aql

I

aqt pearsru oqrt aldoad gdural lqSlw'a;rl depdrara Jo salneq aql lq8g lsmu

(f) pue'(d'(v)

a.{vr

fl

"U[tT

'praua8 ool lnq Suqdual st'1ua1orl 'srauapru8 ueqrnqns lryaread aslmJaqlo uala'auo dlenuassa sr plrorvr aql leql eapr aql'(6) -,{rala qq}l,rl arp uorlnJesrad pue aJuelorl ro1 puualod aql leql g utaod aql Jo aruaq} aql i1

aql'(g) leqt alou

1aso1c

uara lou are

'Z?

'It

'ruaod aq1;o pafqns aqi

aqi;o uouduJsap aql uI pl4ur ol awoJ deru (3) ursruerrela8a,r pue (g) 'uaod aq11o pa[qns rrletu aql uBtI] raq]er IIEpp roultu e $ +I lnq'sauq pu u8rsap adexpuel -g aql rn pauoquau (@ ulearp e sr aJaqJ 'pafqns IerolIT arll lou $ lnq'a)ualorrr lnoqe auaql 8ur,{traprm s}r saleurumlil pue 'uraod aql Jo }xa}qns aql saurp[os (V) slzeN ol aJuara]ar leuq aqJ '$lJnqJpoom Jo uapre8 aql plr o+ slduralle s,raleads aql saqlrrsap dgera111 uraod aql sr Jaqlrau 1nq'uapre8

'fupno7

fl

'0t

uourallor ZL6I raq ruory $ ,,$[Jnqrpoo6,, 'aJII ueqrnqns alqe]ro]ruoJ 1o'ate daql se qons'sqldap aql atnruexa ol spual oqm laod druroduraluoJ paralar e sr urun) aqxpntr ns-0n sNot$ln0 dJ1

'3op Sqr8edruer e Jo uorsrl s,Esrno.1 aopeqsaloJ lourrpJ os pue 't{ent t{uu rn Jesae3 ol JaJaJ }ou saop ,,aleJllapul, 'a8essed aql Surpea,r-Jalo Jo aldtuexa raqloue sr (g) '8uorm are (q) pue (f,) os'dppoqs arB s8u8uolaq sTq leql rou'asreor dlercadsa sr ao{ leqt 1sa33ns o1 3tm11ou sr aJaW '8uorm sr (y) os 'a8essed aq+ Jo uraJuoJ ,{reunrd aqt lou sI Joaraql TJ€l ro dterllap 'ao{ pue esmol uaamlaq sa)uaraJJlp aql srapuod praua8 rn a8essed srul q8noqllv 'suraruo) Fnxas raq ol palrauuor sl aJII raq ol Surrq dew ao{ soeqr aql }noqe uJaJuoJ s,esrnoT }Eql sI araq lurod aqJ 'a}eJIIapuI are spepp I€nxas rrqder8 lalecrlapur are salof d1n6 ,,'pnxas dlalerrdorddeut,, JoJ rusnuaqdna e st ,,alEJIIapuL,

g

'6€

'(d a*t u^rolllerus Io arqes e 1(3) a8ueqr asryar aldoad uaqm suaddeq }eq/vr ar{l pue sr aldoadsurvrol aql Io rorral palel;tn ;o aldruexa ue sr 1q31d pes s,resaeJ la8edruer € uo resae) Jo uorsrl rar{ uI pa}e4suourap are (g) sreal s,eslnol lasealar s,resae3 sa8rn aq uaqn' (y) ptlpe.ld pue pq{ aq o} umot{s sr ao{'alpu8rl r€sa€J aql Jo Euaurqsqdurorce alqeJrpu aql aqlrJsap Euaua}els aq}

[V iI 'B€

'a8essed aq+ peaJ-ralo l,uo6l 'swdap

uappq ol anlJ e lou'alqdrnsap dlprervr 'saop eslnoT ueql dpuaraglp resae3 aas a/vr 1nq ,,'pa1seul,, -ropq8re4s lsnf sr alqca[pe sr-ql aq p1noJ aJuaq pu€ 'snorJrl sr resaeJ 1eq1 uopsaSSns aql sI araql asnp)aq aldoad auros palsFrr azreq 1q3p (g) 'acey sueatu ,,a?esqy,, 'pa8esr,r-ppu sr aq lare; altua8 e s€r{ rcsae)

g 'l€

43'

c

"Darwinian pieties" is an- oxlimoron because it is a contradiction in terms. Darvrrin,s evolutionary theories are seen by many to be antireligio,rri...rr. they conhadict the story of creation in the Bible' At td same time, "p!"ry" ir"un, ..tigious dLvotion. is suggesting that people are devoted to Darwin ur utinaty ur-in"y ever were to religion. Howevel, Darwin is used here not so much to suggesf .rrolrrtion as to suggest the political beliefs associated wirh "social Darwinism,,wirich ,.u, **gg", puoptu u, u violent zero-sum contest for survival in which the r,iinl"., are proven

K;-*

b.#.";;;ilJor

;rffi"Sj:i*,?:il#::il M'

D

45'

c

is orten a polite mask ror racism

ani

to be biologically other social biases ugiir,rt

The tone of the poem contradicts the speaker's attifudes, so nothing in the poem proves those attitudes correct. Ail the other statements about rrne 4 are true. The entire poem is an ironic satire against people who refuse to recognize the violence of their lives as part of the violence of ihe *orta.'1a; *i'1o; ur. tempting because both Ku_ min and presumably the reader feel outrage *," uiolence in the world and would wish to protest it, but the poem itself remains irinic. There is stanza,but its effectivenes, i, ur,J"r.rt by the pathetic There is no helpless sorrow (B) anywhere i" thi

*.r

,:i:ffi.:if;ilflirlt]# ffifJ.^

45'

D

po"J'

The entire point.oj th9 poem is the speaker's growing awareness of his or her own violent tendencies' The idea thit all violence is relatei @) is"impried in the poem, but is not the of the final two lines. The other uru*.r, ur*ll variouslyirn"I"*

ill|"tfft 47'

r"rrreadings of

B

The garden, usually considered a peaceful place, is here the scene of violence, which is iron_ ic' so choice I istrue' The garden ouuio.r"ty ,y-forJ.r which is the site of Darwinian struggle' so II is true' I-n this poem, "u*r., the garden is very realistic and specific, suggesting that the issues discussed u.. ,p..ifi. una ..Xnru. u, rv.il, so IV is true. Iiem III is false, but tempting because gardens are often feafured inpastorJpoems.Ilowevel, pastoral poems focus on the peacefulness of the countryside, which this poem obviously does not. Item V is obviously false, and contradicts the maln theme of tf,. po._. 48' B In the final line of the poem, "Nazi" is an allusion to the atrocities of genocide in world war II' The poem as a whoie implies a comparison between the killing ;;J;h;cks and genocide' but this is merely impiiea and "f (C) or dols not become an outright simile metaphor (A) in the final line' Neither does the final line contain u putuao* @) or metonym (E). If you do not know what these terms mean, you should .onsuli the gtossary at the back of this book. 49' c "Beheading the carrots" is an-example of personification because it applies a physical man qualiry to an inhuman object: Larrots jicr6ing huao",t t urr. t.ads to lop the woodchucks' actions in this wiy makes "if;;; the woodch,r.k, ,ourrd more sinister. Anthropomor_ phism (D) is a more involved form of personific"ri"tt i" *nich one ascribes human motivations to the nonhuman object.

50'

c

The speaker of a poem is not necessarily the poet, but is a narrator. That makes ,,the poet,, (A) the wrong answer. The other answers are pretty ridiculous.

206

f

CRACKII{G THE AP ENGI.ISH TITERATURE EXAM

I0Z

f

SN0ltVNvldXl 0NV SUl/llsNV :Z lstl t)tl)vUd

//'uaasun/tueaJp//

ptrr- ,daap fsdaa1,, aseJ sryl ur 'sarudq,r asnardur a.le saurdqr iuelg ,(pca;rad siry rlJIr{M ,(g) dluo saleal leqg ('urexa arll roJ ,vrou) ol ur,ra1 poo8 raqloue s,+r-,,lear Sururaas;o ftqenb aql,, sueatu apnllF-lx]slJaT$ 'pauopueqe ]ou sr ruaod aql Jo apnllTnursrJa^ aql aJoJararll pue '1ealrns uel.Il JaqlpJ JIISITBaJ sI pauoquaru ruealp aql asneraq Suo.rzvr sr (g) 'ursrrcluauruoJ -IAua lnoqe 1ou'arualorl uerum{ +noqe sI lurod proru 3uo4s s,ruaod aq} asneJaq Suoral, sr

(@ 'ulaod arll Jo pua aql le uol+Jlpe4uoJ ou sr araql asneJaq Suo.rrvr sr (3) 'cruorr dlurepar are sauq Fug aql asn€Jaq Suorzvr q (v) 'go.I qSnorql paraMsue aq ueJ uorlsanb srq;

s

't9

'und e q,,1q8p4u,,1eq1nod plol dpearp p uorlsanb dgeluapoul 2ooq slql Jo {r€q aq} ur dressolS eq1 dprys o} arns aq,srural asarg .trouT },uop nod .(g) uorou -dxo ue 1ou s,ll os !1aqr +Jlperluor lou saop +I 'uorsnlp ue se TJoM o1 q8noua rrprads dgear lou sr lr 1nq'rualsds p8al aql o1 (q) uorsnlle ue se pear aq +qgpr iI .(g) Surq+due azqoqurds lou saop ll puu'(y) .roqdelaru e aq ol q8noua paueisns +ou sr lJ's1JnqJpoom aql Sursse8 /$Irnqrpoom aql Suqncaxa ur pagusnf ;o ssarord prrsdqd aql ol Sur.rra;ar osle +nq i1a; raleads aql teril ,vror{s o+,,aser lq8rpre,,ue lo eapr p8al aql ol Srnrra;ar,rmd e sr,,1q8r14y,,

J

'99

I

'sluaruatels anrl aql qsq dprarror +eqt arror{r dpo aql sr (q) .spnqrpoom uo snJoJ plnoqs ureod srql lo dreurums ou pue 'arnleu uerunq +noqe Jo pealsrn s)Jnr{Jpoorvt +noqe aq ol suaas flns aql ]srrl aqJ 'ruaod aq1;o dreununs e paraprsuo) aq ol uoqerurolur q8noua arrr8 1ou saop atnl +srrJ aq+ asnuJaq'aqey sr A 'an4 sr AJ os 'paueddeq dlear leqrvt;o luaualelsJaprm ue q ,,1q31,r lno uJnl l,uplp,, +eql lvrou)I nod'ruaod aql Jo pua aq1 o11a3 nod aunl aql dg 'an.4 osle $ pue luaua]e]s se 8tryp aru€s ar{l dpcexa lsorup des o1 asrnorsrp Jnuape)e alol.u e sasn 11I ]uaura+els ZsaJn+eaJJ Surlq Sursse8 are nod uaq.,vr 8uor,u sr ]Eq1vr

I

1nq,,€uo.rn, +uam s8qql lBt{+ saudur ,,lL13g lno rnnl l,upIp,,1an.4 sr II iuarua}els .$lrnr{J -poom atll Sursse8 lnoqe drols E reaq ol Suro8 ale nod 1eq1 nod sllal auq lsrr; aql 1an4 sr 1 luauale+s 'asleJ Jo an4 se +uarualels qJea al€nle^a 01 $ uoqsanb snfi JaMsue oi dem aq1

o'29

'pasryuoJ l,usr raleads aql pue ssalasuas l,uq aurT aql asneJaq Suo.rrvr lno +elJ 1sn[ s (g) 'craodoleruouo dgenadsa 1ou sr 1r 'anrleral1le q aql ar{l q8noqlp asneJaq Suo.rrvr sr (q) 'sauq snor,rard rr arolaq auo8 a,req sprom qsreq dlenba 1arn1 sIIl ol anblm lou sr paJJa aq+ +nq'pron .realc dlqsrer{ e sr ,,rallD{,, asneJaq ,an4 dlplped sr (l) 'lqod srql dlpexa lswe8e Surn€re $ aloqtvr e se uaod aq+ lnq'd.rolepard dlernleu ore sueumq l€ql splor{ rusnn.trreq proos 1d8o1oapr Jo aruanlJul arp Jo asneraq Suqdrual sr (g) 'sarSoloapr IZeN Jo aJuanlJul arp tn se 'ayqeldarre sarlrJo4e Japual os pue dlanos uerunq rn srrorpe plruq dueur azqemleu sldurs q)ns tErD uo4saS8ns E ua^a sr araqJ dplnrq os pe ol ra>1eads aql Sutcuangtn ldrlrs IErnlTnJ I€rJrJqrE aql Suzrser{drua sr laod aq1 'auo)s pa8els e Io eapr aql ol suapre8 prnleu;o 3u41as e ruory Stqnplrms dg dpr4 alltl e s uoqsanb sIqJ

V 'Ig

HOW TO SCORE PRACTICE TEST 2 Section f: Mulfipfe Choice

Secfion (See

_*______)

*

number correct

number wrong

fl: Free Response

if you can find a teacher or classmate

+

essay

1

(out of 9)

Multiple-Choice Score

+

3

essay 2

essay

(out of 9)

(out of 9)

.

tro

score your essays using the guidelines in Chapter Z .)

------

Free-Response Score

(omposile Score Multiple-Choice Score

Free-Response Score

Weighted Section I Score

x

1.23 = Weighted Section

3.06

I

Score

II

Score

=

Weighted Section

Weighted Section

AP Grode Conversion

208

I

CRACKING IHE AP EiIGI.ISH TITERATURE EXAM

I

Score

Composite Score

rfuossolC

Related Documents

Ap English Test
November 2019 8
Ap English
May 2020 6
English Test
June 2020 10
English Test
May 2020 12
English Test
July 2020 12
English Test
June 2020 9